Origin of Universe Qs for Atheists

Aedus
Posts: 33
Joined: 2009-08-07
User is offlineOffline
Origin of Universe Qs for Atheists

What are your thoughts on the origins of the universe? I guess most atheists would subscribe to some sort of cyclical multiverse theory, in which case universes & galaxies are constantly being created throughout all eternity. But is this an adequate explanation? I'm aware that most atheists will reply with "I don't know," but hopefully we can extrapolate.

Can an infinite number of universes be quantified or rationalized? Why should the "default" state of the multiverse be one that creates galaxies and then life? Even if the universe existed for perpetuity, motion is not a property of matter, so what got the multiverse moving in the first place? Theists would say this thing is God. Why can't it be God, or, what do atheists offer instead of God?


Vastet
atheistBloggerSuperfan
Vastet's picture
Posts: 13234
Joined: 2006-12-25
User is offlineOffline
Actually, most atheists will

Actually, as you point out, most atheists will simply say they don't know, but that there are a number of interesting theories that have yet to be proved or disproved. Speculation doesn't accomplish anything.

Enlightened Atheist, Gaming God.


cervello_marcio
Superfan
cervello_marcio's picture
Posts: 210
Joined: 2009-05-19
User is offlineOffline
multiverse theory is not the

multiverse theory is not the default atheist position just b/c dawkins mentioned it in god delusion

 

"Do not, as some ungracious pastors do, show me the steep and thorny way to heaven. Whiles, like a puff'd and reckless libertine, himself the primrose path of dalliance treads. And recks not his own rede."


Subdi Visions
Bronze Member
Subdi Visions's picture
Posts: 278
Joined: 2007-10-29
User is offlineOffline
Aedus wrote:Why can't it be

Aedus wrote:
Why can't it be God, or, what do atheists offer instead of God?

Because your god is make believe.

What god created your god? And what god created that god? Ad nauseum, ad infinitum...

Why is it possible for there to be a sentient, all powerful creator of life that spontaneously popped into existence with no creator but the universe itself can't have?

Your god equation makes no sense. A child can see the faulty logic until they've been indoctrinated into seeing the emperor's new clothes...

Respectfully,
Lenny

"The righteous rise, With burning eyes, Of hatred and ill-will
Madmen fed on fear and lies, To beat and burn and kill"
Witch Hunt from the album Moving Pictures. Neal Pert, Rush


Kavis
atheist
Kavis's picture
Posts: 191
Joined: 2008-04-17
User is offlineOffline
I know it's a bit impolite

I know it's a bit impolite to answer questions with more questions, but: Why would most atheists ascribe to a cyclical multiverse worldview? Also, why would a multiverse necessarily imply an infinite number of universes? How would one even attempt to quantify an infinite number of universes anyway?

I accept the Big Bang Theory as an accurrate history of the universe so far.  The evidence is overwhelming.  That said, the BBT doesn't speak to origins.  It marks the earliest event we know about (and possibly can know about, as concepts of space/time get somewhat odd before the first moment of expansion). 

During the earliest parts of the Big Bang, there was no matter.  Everything that is and ever was and ever will be in this universe existed as energy.  It wasn't until the universe expanded enough, and thus cooled enough, that matter began to congeal from the radiation and light.  It was still later that matter began taking on the properties we know now, such as protons and neutrons clumping together into nuclei, gaining electrons, and dividing into regions of dense matter and relatively empty space.  So, starting with the expansion of space, the titanic amount of energy sloshing back and forth, and the basic forces at work upon energy and matter, why do we need a God to nudge the first bits of matter around? Remember that gravity is the WEAKEST of the elemental forces, and it alone is enough to form black holes, galaxies, and wind from matter. At the scale of the early Big Bang, the strong and weak nuclear forces and the electromagnetic force would have been (and indeed were) strong enough to whip matter around hard enough to prevent atoms from forming. Again, where do we need a God to explain any of this? That's to say nothing of quantum waveforms and Brownian motion.

Perhaps you mean before then, specifically with your question about motion.  What set the Big Bang off? I am not a string theory or M-theory specialist, but from what I've seen we don't know.  Yes, it's alright not to know, and we don't need to put a sign at the edge of the map of history proclaiming HERE THERE BE GODS.

Why do we need no such sign, no God of the gaps? We don't need them because they don't explain anything.  Tell me, if you accept God as that which set the Big Bang in motion, what do you know by that hypothesis? What predictions can you make about the nature of reality, about present and future events? What number of those predictions can you actually test and falsify?  How does the God hypothesis explain evidence that is currently baffling our best minds and most sensitive instruments? What good is God?

Religion is a virus.
Fight the infection.


Brian37
atheistSuperfan
Brian37's picture
Posts: 16433
Joined: 2006-02-14
User is onlineOnline
This is such an absurd

This is such an absurd question.

The answer is simple., "We don't know". You don't nor do we.

Which causes the believer to default to believer propaganda, "So then you are not an atheist, but an agnostic"

BULLSHIT!

An atheist is one who does not hold a belief in a god or gods.

THE DIFFERENCE, is when the atheist says "I don't know" we don't default to magic, superstition or ancient myth to fill in the gap. But that does not mean we sit on the fence.

I would highly suspect that whatever went into the process that we see today is as natural (all be it not yet understood, and may never be), but in no way constitutes inserting myth, superstition, fantastic claims, or magic for any gap answer.

Saying "I don't know" only makes me an agnostic by proxy of theist belief, it still does not change that all the claims of the super natural are as credible as claiming Harry Potter can fly around on a broom.

I am an atheist. I do not buy any claims of supernatural, past or present. Because I don't know the future does not negate my current position. My current position is that gods are nothing but a product of human imagination.

 

 

"We are a nation of Christians and Muslims, Jews and Hindus -- and nonbelievers."Obama
Check out my poetry here on Rational Responders Like my poetry thread on Facebook under Brian James Rational Poet, @Brianrrs37 on Twitter and my blog at www.brianjamesrationalpoet.blog


BobSpence
High Level DonorRational VIP!ScientistWebsite Admin
BobSpence's picture
Posts: 5939
Joined: 2006-02-14
User is offlineOffline
Kavis has explained it

Kavis has explained it pretty well.

To also comment on the motion question, that's seems to hark back to the medieval idea of the 'need' for a 'first mover'.

To give some simple examples of how motion does not need some external intervention.

Imagine two physical objects separated from each other. In the presence of gravity, rather than needing something to start them moving, they would require external intervention to stop them from moving

If they are motionless with respect to each other at some point, they will start moving towards each other at an increasing rate, collide, and bounce off each other, and continually keep bouncing like that until the energy dissipates as heat, which is just the movement of the atoms of which they are composed.

Two electrons, or two protons, or other sets of particles with the same polarity of charge, will need other forces to stop them moving away from each other under the influence of electric fields.

So the 'first mover' is either a field, or a property of space-time, as in Einstein's Theory of Gravity.

Now as to what started things happening, we don't know.

Quantum Theory, which is well-supported by observation and experiment, seems to show that even 'empty space' cannot have absolutely zero energy. That would be a violation of the Uncertainty principle, in that the energy state of anything has minimum level of 'fuzziness', what seems to be be a minimum level of random energy, which is all that is required to allow a finite probability that eventually there will be a fluctuation of energy large enough to pass some threshhold to trigger what we would see as the initial state of the Big Bang.

Where would such an 'energy field' 'come from'? Same place 'God' came from. Or maybe it was just always there, just like 'God'.

What seems easier to imagine just coming into existence? A minimal, featureless field of the closest to 'nothing' that can possibly be, or an infinite sentient super-being?

Neither the Big Bang theory or God is an ultimate explanation for existence. We can at least suggest plausible scenarios (ie based on minimal extensions of current scientific understanding), for the emergence of our Universe, given the absolutely minimum initial state of 'something' existing.

Inserting God into the picture offers no ultimate explanation, just adds something inherently incomprehensible and unnecessary into the picture.

Even if you want to claim something was needed to start things going, you still have to justify why it would have any of the typical attributes of God, such as being a conscious being, of infinite size and power, etc. 

So Theism has a bigger problem than Atheism to explain where everything originated, because it adds an vastly more incomprehensible entity into existence while avoiding or ignoring the question of explaining where it 'came from'.

 

Favorite oxymorons: Gospel Truth, Rational Supernaturalist, Business Ethics, Christian Morality

"Theology is now little more than a branch of human ignorance. Indeed, it is ignorance with wings." - Sam Harris

The path to Truth lies via careful study of reality, not the dreams of our fallible minds - me

From the sublime to the ridiculous: Science -> Philosophy -> Theology


Aedus
Posts: 33
Joined: 2009-08-07
User is offlineOffline
cervello_marcio

cervello_marcio wrote:
multiverse theory is not the default atheist position just b/c dawkins mentioned it in god delusion

I never said it was the default.

 

Subdi Visions wrote:
Because your god is make believe.

I'm not a Christian.

 

Subdi Visions wrote:
What god created your god? And what god created that god? Ad nauseum, ad infinitum...

Why is it possible for there to be a sentient, all powerful creator of life that spontaneously popped into existence with no creator but the universe itself can't have?

Theists believe that the laws of logic i.e. cause & effect come from a creator, and as such he is unbounded by these rules, making him the uncaused cause. In short, he's the exception.

 

Subdi Visions wrote:
Your god equation makes no sense. A child can see the faulty logic until they've been indoctrinated into seeing the emperor's new clothes...

I have a theory that doesn't contradict itself or butcher Occam's Razor. Frankly, I like it better than "the universe just always existed, end of story" or "the universe popped out of literally nothing & here we are". If you feel that the last 2 examples are somehow more logical than the first then that's your prerogative. <_<

 

Kavis wrote:
but: Why would most atheists ascribe to a cyclical multiverse worldview?

The universe popping out of literally nothing i.e. Big Bang would point to a creator outside of time/space. A cyclical universe has no need for a God because it always existed.

 

Kavis wrote:
Also, why would a multiverse necessarily imply an infinite number of universes?

I didn't say it did. (if I did, then I didn't mean to)

 

Kavis wrote:
How would one even attempt to quantify an infinite number of universes anyway?

I have no idea - it makes no sense, which is why I think our universe/multiverse had a definite beginning.

 

Kavis wrote:
Perhaps you mean before then, specifically with your question about motion.  What set the Big Bang off? I am not a string theory or M-theory specialist, but from what I've seen we don't know.  Yes, it's alright not to know, and we don't need to put a sign at the edge of the map of history proclaiming HERE THERE BE GODS.

Indeed, but at some point you have to put your beliefs under rational scrutinity. This idea that "wel'll never know" is simply not true. Europeans once thought that it was impossible to know what was on the other side of the atlantic ocean. Our scientific knowledge is still in its infant stages, and we won't be able to just say "I don't know" forever. Also, I'm not asking you for details on how it works - I doubt any of us are astrophysicists. I'm just inquiring whether the basic philosophy is logical/rational. Furthermore, string theory and the like does not have to be cyclical - the universe could still have had a beginning.

 

Quote:
Why do we need no such sign, no God of the gaps? We don't need them because they don't explain anything.  Tell me, if you accept God as that which set the Big Bang in motion, what do you know by that hypothesis? What predictions can you make about the nature of reality, about present and future events? What number of those predictions can you actually test and falsify?  How does the God hypothesis explain evidence that is currently baffling our best minds and most sensitive instruments? What good is God?

I don't concern myself with that; it's the job of science to make predictions. Truth cannot contradict truth. My beliefs do not impact my life here on Earth whatsoever - I am as "free" as any atheist.

 

Brian37 wrote:
"So then you are not an atheist, but an agnostic"

BULLSHIT!

An atheist is one who does not hold a belief in a god or gods.

Atheism has always meant "godless belief" as indicated by its etymology. Just because a few atheists decided to expand the definition to include more people and so that they would have the edge in debates does not change this. A word is defined by common usage, not by the people who fit that definition.

 

Quote:
To also comment on the motion question, that's seems to hark back to the medieval idea of the 'need' for a 'first mover'.

To give some simple examples of how motion does not need some external intervention.

Imagine two physical objects separated from each other. In the presence of gravity, rather than needing something to start them moving, they would require external intervention to stop them from moving

If they are motionless with respect to each other at some point, they will start moving towards each other at an increasing rate, collide, and bounce off each other, and continually keep bouncing like that until the energy dissipates as heat, which is just the movement of the atoms of which they are composed.

Two electrons, or two protons, or other sets of particles with the same polarity of charge, will need other forces to stop them moving away from each other under the influence of electric fields.

So the 'first mover' is either a field, or a property of space-time, as in Einstein's Theory of Gravity.

Aye, I don't disagree with that - I simply think that this "first mover" is another one of the assumptions that go into a cyclical multiverse that supports life. It adds to the improbability of such a universe being the only one to exist for eternity (the universe must now support a "first mover&quotEye-wink.

 

BobSpence1 wrote:
Now as to what started things happening, we don't know.

Quantum Theory, which is well-supported by observation and experiment, seems to show that even 'empty space' cannot have absolutely zero energy. That would be a violation of the Uncertainty principle, in that the energy state of anything has minimum level of 'fuzziness', what seems to be be a minimum level of random energy, which is all that is required to allow a finite probability that eventually there will be a fluctuation of energy large enough to pass some threshhold to trigger what we would see as the initial state of the Big Bang.

Where would such an 'energy field' 'come from'? Same place 'God' came from. Or maybe it was just always there, just like 'God'.

What seems easier to imagine just coming into existence? A minimal, featureless field of the closest to 'nothing' that can possibly be, or an infinite sentient super-being?

The quantum fluctuations you describe are part of an infinite universe. We can assume that an infinite series of quantum fluctuations produced an infinite number of universes already, which I have hard time wrapping my head around. We haven't encountered anything truly infinite other than in concept (well, depending on what theories you subscribe to, the center of a black hole has infinite density, but AFAIK most physicists think that the center of a black hole will eventually yield a finite mass).

 

Quote:
So Theism has a bigger problem than Atheism to explain where everything originated, because it adds an vastly more incomprehensible entity into existence while avoiding or ignoring the question of explaining where it 'came from'.

Atheists don't have a monopoly over "I don't know," and I wouldn't say that atheism offers any better alternatives than theism. As I said before, I have an explanation that jives & doesn't

 


Brian37
atheistSuperfan
Brian37's picture
Posts: 16433
Joined: 2006-02-14
User is onlineOnline
BobSpence1 wrote:Kavis has

BobSpence1 wrote:

Kavis has explained it pretty well.

To also comment on the motion question, that's seems to hark back to the medieval idea of the 'need' for a 'first mover'.

To give some simple examples of how motion does not need some external intervention.

Imagine two physical objects separated from each other. In the presence of gravity, rather than needing something to start them moving, they would require external intervention to stop them from moving

If they are motionless with respect to each other at some point, they will start moving towards each other at an increasing rate, collide, and bounce off each other, and continually keep bouncing like that until the energy dissipates as heat, which is just the movement of the atoms of which they are composed.

Two electrons, or two protons, or other sets of particles with the same polarity of charge, will need other forces to stop them moving away from each other under the influence of electric fields.

So the 'first mover' is either a field, or a property of space-time, as in Einstein's Theory of Gravity.

Now as to what started things happening, we don't know.

Quantum Theory, which is well-supported by observation and experiment, seems to show that even 'empty space' cannot have absolutely zero energy. That would be a violation of the Uncertainty principle, in that the energy state of anything has minimum level of 'fuzziness', what seems to be be a minimum level of random energy, which is all that is required to allow a finite probability that eventually there will be a fluctuation of energy large enough to pass some threshhold to trigger what we would see as the initial state of the Big Bang.

Where would such an 'energy field' 'come from'? Same place 'God' came from. Or maybe it was just always there, just like 'God'.

What seems easier to imagine just coming into existence? A minimal, featureless field of the closest to 'nothing' that can possibly be, or an infinite sentient super-being?

Neither the Big Bang theory or God is an ultimate explanation for existence. We can at least suggest plausible scenarios (ie based on minimal extensions of current scientific understanding), for the emergence of our Universe, given the absolutely minimum initial state of 'something' existing.

Inserting God into the picture offers no ultimate explanation, just adds something inherently incomprehensible and unnecessary into the picture.

Even if you want to claim something was needed to start things going, you still have to justify why it would have any of the typical attributes of God, such as being a conscious being, of infinite size and power, etc. 

So Theism has a bigger problem than Atheism to explain where everything originated, because it adds an vastly more incomprehensible entity into existence while avoiding or ignoring the question of explaining where it 'came from'.

 

YOU SON OF A BITCH.....HOW DARE YOU BURST THEIR BUBBLE!

If they want to believe that all this is a result of a magical thinking manipulator, who do you think you are?

Next thing you are going to tell me is that Santa isn't real! YOU BASTARD!

SANTA IS REAL BECAUSE I GOT MY GI JOE HELICOPTER UNDER THE TREE WHEN I WAS 7!

"We are a nation of Christians and Muslims, Jews and Hindus -- and nonbelievers."Obama
Check out my poetry here on Rational Responders Like my poetry thread on Facebook under Brian James Rational Poet, @Brianrrs37 on Twitter and my blog at www.brianjamesrationalpoet.blog


Kavis
atheist
Kavis's picture
Posts: 191
Joined: 2008-04-17
User is offlineOffline
Wow.  I think this is about

Wow.  I think this is about the most thorough reply I've seen on this forum.  Kudos. I'll concern myself with just some of it:

Aedus wrote:

The universe popping out of literally nothing i.e. Big Bang would point to a creator outside of time/space. A cyclical universe has no need for a God because it always existed.

The Big Bang doesn't imply the Universe popped out of nothing, it implies a change of state. The universe can be eternal without being cyclical.  Phil Plait's Death From the Skies! has some interesting conjectures about the distant future of our universe, and the Big Crunch model has been pretty thoroughly abandoned.

 

Quote:

I didn't say it did. (if I did, then I didn't mean to)

I was referring to the first question of the second paragraph of your first post.  Subsection 3.

 

Quote:

I have no idea - it makes no sense, which is why I think our universe/multiverse had a definite beginning.

That was kind of a trick question.  You can only attempt to quantify an infinity for as long as you're willing to sit there and do it.  It's, by nature, a futile task.

 

Quote:

Indeed, but at some point you have to put your beliefs under rational scrutinity. This idea that "wel'll never know" is simply not true. Europeans once thought that it was impossible to know what was on the other side of the atlantic ocean. Our scientific knowledge is still in its infant stages, and we won't be able to just say "I don't know" forever. Also, I'm not asking you for details on how it works - I doubt any of us are astrophysicists. I'm just inquiring whether the basic philosophy is logical/rational. Furthermore, string theory and the like does not have to be cyclical - the universe could still have had a beginning.

We may never know.  I didn't mean to imply that the state of the universe before the instance of the BB was unknowable (though I freely admit I have asserted that in the past, and I know better than to make knowledge claims about the unknowable).  The basic philosophy is rational - perhaps not perfectly so, perhaps some people hold it in an irrational fashion, but it is rational to suspend belief about the origin of the universe until presented with evidence.  I'll again admit that nobody I know has ever presented such evidence.  The difference between science and religion is that science might present such evidence while religion can only present claims about beings to whom the concept of evidence does not apply.

Quote:
I don't concern myself with that; it's the job of science to make predictions. Truth cannot contradict truth. My beliefs do not impact my life here on Earth whatsoever - I am as "free" as any atheist.

You don't get to excuse yourself from epistemology just because you're a religious person rather than an irreligious one.  Without predictive power, evidence and tests, what do your knowledge claims mean? What can you claim to know at all if those knowledge claims are exempt from reflecting concrete reality?

 

Religion is a virus.
Fight the infection.


Vastet
atheistBloggerSuperfan
Vastet's picture
Posts: 13234
Joined: 2006-12-25
User is offlineOffline
Frankly, the laws of physics

Frankly, the laws of physics would support an infinite cyclical universe. Getting your head around an idea is hardly a requirement for truth. Quantum physics has conclusively proven that there is much about existence which is counter-intuitive.
Which is not to say it's how it happened, merely that there are possible explanations that fit with what we know, and no religion can claim to hold any of them.

Enlightened Atheist, Gaming God.


Tapey
atheist
Tapey's picture
Posts: 1478
Joined: 2009-01-23
User is offlineOffline
Tbh i find the whole origns

Tbh i find the whole origns thing kinda boring, we are here and i don't care how it happened, only that it did. Im glad people are looking into it though, im sure if they ever find it, they will find some way to make the knowledge usefull, but untill that day comes i don't really care and won't spend to long wondering. Unless there is a practical benefit i don't see why I should care either.

Whatever goes upon two legs is an enemy.
Whatever goes upon four legs, or has wings, is a friend.
No animal shall wear clothes.
No animal shall sleep in a bed.
No animal shall drink alcohol.
No animal shall kill any other animal.
All animals are equal.


BobSpence
High Level DonorRational VIP!ScientistWebsite Admin
BobSpence's picture
Posts: 5939
Joined: 2006-02-14
User is offlineOffline
Aedus wrote:Quote:To also

Aedus wrote:

Quote:
To also comment on the motion question, that's seems to hark back to the medieval idea of the 'need' for a 'first mover'.

To give some simple examples of how motion does not need some external intervention.

Imagine two physical objects separated from each other. In the presence of gravity, rather than needing something to start them moving, they would require external intervention to stop them from moving

If they are motionless with respect to each other at some point, they will start moving towards each other at an increasing rate, collide, and bounce off each other, and continually keep bouncing like that until the energy dissipates as heat, which is just the movement of the atoms of which they are composed.

Two electrons, or two protons, or other sets of particles with the same polarity of charge, will need other forces to stop them moving away from each other under the influence of electric fields.

So the 'first mover' is either a field, or a property of space-time, as in Einstein's Theory of Gravity.

Aye, I don't disagree with that - I simply think that this "first mover" is another one of the assumptions that go into a cyclical multiverse that supports life. It adds to the improbability of such a universe being the only one to exist for eternity (the universe must now support a "first mover&quotEye-wink.

"First mover", in respect of actual motion is an old concept arising when it was thought that objects actually needed some external force to keep them moving, that the 'natural state' of an object was to remain 'at rest' unless actively being 'moved'.

Galileo, Newton, and Einstein render such ideas completely obsolete.

Quantum Theory renders the need for specific initiating causes questionable, at the very least.

So bringing "first mover' ideas into a serious discussion is not particularly useful.

In any case, there is nothing about a cyclic universe or life that is particularly relevant to 'first cause' ideas in modern understanding.

There is a current theory that there may be a succession of Universes that is not actually cyclic, which is much more consistent with current observations that seem to point to an accelerating rate of expansion, rather than an eventually slowing and collapse. It envisages that at some point when individual bits of matter/energy have become so spread out in the accelerating expansion, with dark energy which seems to be associated with the acceleration and becomes increasingly dominant, can trigger the formation of new 'Big Bang' events from the remaining fragments of the original universe. 

Quote:

BobSpence1 wrote:
Now as to what started things happening, we don't know.

Quantum Theory, which is well-supported by observation and experiment, seems to show that even 'empty space' cannot have absolutely zero energy. That would be a violation of the Uncertainty principle, in that the energy state of anything has minimum level of 'fuzziness', what seems to be be a minimum level of random energy, which is all that is required to allow a finite probability that eventually there will be a fluctuation of energy large enough to pass some threshhold to trigger what we would see as the initial state of the Big Bang.

Where would such an 'energy field' 'come from'? Same place 'God' came from. Or maybe it was just always there, just like 'God'.

What seems easier to imagine just coming into existence? A minimal, featureless field of the closest to 'nothing' that can possibly be, or an infinite sentient super-being?

The quantum fluctuations you describe are part of an infinite universe. We can assume that an infinite series of quantum fluctuations produced an infinite number of universes already, which I have hard time wrapping my head around. We haven't encountered anything truly infinite other than in concept (well, depending on what theories you subscribe to, the center of a black hole has infinite density, but AFAIK most physicists think that the center of a black hole will eventually yield a finite mass).

Quantum fluctuations neither require or imply an infinite universe.

Quote:

Quote:
So Theism has a bigger problem than Atheism to explain where everything originated, because it adds an vastly more incomprehensible entity into existence while avoiding or ignoring the question of explaining where it 'came from'.

Atheists don't have a monopoly over "I don't know," and I wouldn't say that atheism offers any better alternatives than theism. As I said before, I have an explanation that jives & doesn't

 

No, Theism postulates an intrinsically more complex thing, with attributes not following from proper arguments, and which is simply unnecessary, so it is nonsense to say it is a better 'alternative'. 

It is not really an alternative, it is simply throwing in an extra entity which ultimately does not add anything to explain its own origin, let alone the origin of anything else.

It is nearly as stupid as the argument which tries to argue that the Laws of Logic were 'created' by a God, which totally misunderstands what 'Logic' is, and how the Laws of Logic are derived.

Favorite oxymorons: Gospel Truth, Rational Supernaturalist, Business Ethics, Christian Morality

"Theology is now little more than a branch of human ignorance. Indeed, it is ignorance with wings." - Sam Harris

The path to Truth lies via careful study of reality, not the dreams of our fallible minds - me

From the sublime to the ridiculous: Science -> Philosophy -> Theology


Subdi Visions
Bronze Member
Subdi Visions's picture
Posts: 278
Joined: 2007-10-29
User is offlineOffline
Use lots of big words, it'll convince us for sure...

Aedus wrote:

Subdi Visions wrote:
Because your god is make believe.

I'm not a Christian.

I didn't say or imply which flavor your god was. It doesn't matter because they're all make believe. Even if you've created your very own personal god. Its make believe as well...

Aedus wrote:

Subdi Visions wrote:
What god created your god? And what god created that god? Ad nauseum, ad infinitum...

Why is it possible for there to be a sentient, all powerful creator of life that spontaneously popped into existence with no creator but the universe itself can't have?

Theists believe that the laws of logic i.e. cause & effect come from a creator, and as such he is unbounded by these rules, making him the uncaused cause. In short, he's the exception.

The exception... And that's OK with your logic? Seriously...?

 

Aedus wrote:

Subdi Visions wrote:
Your god equation makes no sense. A child can see the faulty logic until they've been indoctrinated into seeing the emperor's new clothes...

I have a theory that doesn't contradict itself or butcher Occam's Razor. Frankly, I like it better than "the universe just always existed, end of story" or "the universe popped out of literally nothing & here we are". If you feel that the last 2 examples are somehow more logical than the first then that's your prerogative. <_<

I don't see a first example to ponder over and be in awe of...

The universe doesn't need you to believe that it's always existed and neither do I.

Believing in magical beings that are all powerful yet magically created themselves and don't need to exist within the reality they supposedly created is just silly. But if it works for ya...

 

Respectfully,
Lenny

"The righteous rise, With burning eyes, Of hatred and ill-will
Madmen fed on fear and lies, To beat and burn and kill"
Witch Hunt from the album Moving Pictures. Neal Pert, Rush


latincanuck
atheist
latincanuck's picture
Posts: 2038
Joined: 2007-06-01
User is offlineOffline
Well a small survery

Amongst my atheists friends (6 that responded) was don't know, but the Big bang theory (BBT) with all the evidence is as close to the begining of this universe that we have to explain. All other hypothesis or theories or concepts and or ideas aren't proven yet and therefore cannot be taken as true. With the one thing being that the god described by christianity/judaism, islam, hinduism, and various other religons and mythologies cannot be believed because they cannot be properly explained, nor can the origins of such said gods be properly explained.


butterbattle
ModeratorSuperfan
butterbattle's picture
Posts: 3945
Joined: 2008-09-12
User is offlineOffline
Hello, Aedus, welcome to the

Hello, Aedus, welcome to the forum.

 

Quote:
Can an infinite number of universes be quantified or rationalized?

I can't imagine how anything can be infinite, so my answer is 'I don't know' as well.

 

Quote:
Even if the universe existed for perpetuity, motion is not a property of matter, so what got the multiverse moving in the first place?

Why does it have to begin at rest?

Quote:
Theists would say this thing is God. Why can't it be God, or, what do atheists offer instead of God?

Again, of course, my answer is "I don't know," but I have many issues with the definition of God.

Aedus wrote:
so that they would have the edge in debates does not change this.

Hardly. The position is the same. We just use different words.

 

 

Our revels now are ended. These our actors, | As I foretold you, were all spirits, and | Are melted into air, into thin air; | And, like the baseless fabric of this vision, | The cloud-capped towers, the gorgeous palaces, | The solemn temples, the great globe itself, - Yea, all which it inherit, shall dissolve, | And, like this insubstantial pageant faded, | Leave not a rack behind. We are such stuff | As dreams are made on, and our little life | Is rounded with a sleep. - Shakespeare


Subdi Visions
Bronze Member
Subdi Visions's picture
Posts: 278
Joined: 2007-10-29
User is offlineOffline
Definitions...

Aedus wrote:
Atheism has always meant "godless belief" as indicated by its etymology. Just because a few atheists decided to expand the definition to include more people and so that they would have the edge in debates does not change this. A word is defined by common usage, not by the people who fit that definition.

The argument over definitions has also been going on a long time and has thoroughly been covered by much greater minds already. Give me the name of a person that doesn't believe in astrology please. How about the formal name of someone that doesn't believe in fairies... The formal title of someone that isn't a fan of bowling...

theists gave us the title atheist by their common usage. We don't want to play the silly "let's play make believe" game. As far as I'm concerned you can shove the name of atheist so far up the asses of theists that they wake up out of their stupidity. Frick, frackin slippery fools.

Respectfully,
Lenny

"The righteous rise, With burning eyes, Of hatred and ill-will
Madmen fed on fear and lies, To beat and burn and kill"
Witch Hunt from the album Moving Pictures. Neal Pert, Rush


Aedus
Posts: 33
Joined: 2009-08-07
User is offlineOffline
Before we continue I feel

Before we continue I feel that I should clarify my stance:

Quote:
No, Theism postulates an intrinsically more complex thing, with attributes not following from proper arguments, and which is simply unnecessary, so it is nonsense to say it is a better 'alternative'.

It is not really an alternative, it is simply throwing in an extra entity which ultimately does not add anything to explain its own origin, let alone the origin of anything else.

The evidence is creation, the conclusion is a creator. I make no claims about any of his properties, other than that he is the uncaused cause. We know that simple matter cannot be the cause of its own existence, so we must find something that can. My conclusion follows from the evidence because there are many laws of creation which defy any natural need for their existence, such as the Planck Constant (if it was higher or lower than .00000001 then life could not exist as we know it, and there is no reason for it not to be any higher or lower), the strong force (if it was a trillionth of a centimeter or less then all of the universe's hydrogen would be helium and life could not exist) and the speed of light (if it was any higher or lower life could not have formed), among other things. There is no reason for these constants to be what they are. The odds of such a universe forming are so small that they should basically be zero, which points to a creator. It's deductive reasoning.

But then, if you subscribe to a cyclical multiverse theory which has generated an infinite # of universes throughout eternity, then the odds don't matter and such a universe would eventually be created. Here are the assumptions that go into this:

1) That the constants mentioned above are modified every time a universe is created (whether the universe is created through quantum fluctuations or collision of branes, etc. etc.) As far as I know, there is no science to support this and it's a huge assumption.

2) That this multiverse is eternal and not created. No part of science requires cyclical multiverses to be eternal, and claiming that it is eternal only presents us with more problems i.e. having to cross an infinity, which, as Kavis basically said, is a futile task.

The universe is either created or eternal. I've presented why I think that it's the former, so what is the reasoning for believing that it's the latter? It seems to me that the latter is nothing more than a horrific violation of Occam's razor, supported neither by science nor reasoning. It's like grasping at straws. No matter what most atheists say, they have taken up a position which requires them to support the latter regardless of whether or not it's actually true.

 
Kavis wrote:
You don't get to excuse yourself from epistemology just because you're a religious person rather than an irreligious one.  Without predictive power, evidence and tests, what do your knowledge claims mean? What can you claim to know at all if those knowledge claims are exempt from reflecting concrete reality?


I still don't understand why I'm compelled to draw anything from my beliefs. If I want predictive power then I turn to science.

Vastet wrote:
Frankly, the laws of physics would support an infinite cyclical universe. Getting your head around an idea is hardly a requirement for truth. Quantum physics has conclusively proven that there is much about existence which is counter-intuitive.


I can accept things such as particle-wave duality once they already exist - they are properties of this universe. This and other strange laws of nature defy any natural or sufficient reasons for their existence, which points to an intelligent creator. No branch of science deals with whether the universe is eternal or not, and from what I've seen I'm not compelled to believe that it's the former. Also, science for all its accomplishments still has some limitations in the here & now. What it can't directly measure it ignores. For example, though memory has a basis in biology, is it safe to assume that biology is the only force in work there?

BobSpence1 wrote:
"First mover", in respect of actual motion is an old concept arising when it was thought that objects actually needed some external force to keep them moving, that the 'natural state' of an object was to remain 'at rest' unless actively being 'moved'.

Galileo, Newton, and Einstein render such ideas completely obsolete.


You misunderstand me. I'm saying that motion is not a property of matter and without motion the universe would just be a giant blob of matter making the existence of solar systems and galaxies and life as we know it impossible. It's existence is another one of the assumptions that go into the eternal universe that atheists require, despite there being no reason for it.

BobSpence1 wrote:
Quantum fluctuations neither require or imply an infinite universe.


I didn't make myself clear - I meant a multiverse with an infinite # of universes.

Quote:
Quantum Theory renders the need for specific initiating causes questionable, at the very least.


As would any cyclic universe theory.

Quote:
It is nearly as stupid as the argument which tries to argue that the Laws of Logic were 'created' by a God, which totally misunderstands what 'Logic' is, and how the Laws of Logic are derived.


I was by no means referring to them in the philosophical sense - that wouldn't make any sense.

Subdi Visions wrote:
I didn't say or imply which flavor your god was. It doesn't matter because they're all make believe. Even if you've created your very own personal god. Its make believe as well...

Interesting. Do you consider all deductive inferences to be "make believe" as well? If so then I guess that yes, my god is make believe. Just like the idea of extraterrestrial life - we haven't seen it and have no proof for it, therefore it's make believe.

 

Subdi Visions wrote:
The exception... And that's OK with your logic? Seriously...?

Exception to the rules. It's really a simple concept. If we assume that a Creator exists, then we can assume that he has enough power to bring himself into being. God can be an uncaused cause - simple matter cannot.

Subdi Visions wrote:
The universe doesn't need you to believe that it's always existed and neither do I.

Irrelevant.

Subdi Visions wrote:
Believing in magical beings that are all powerful yet magically created themselves and don't need to exist within the reality they supposedly created is just silly. But if it works for ya...

I find it better than "the universe just always existed - end of story" or "the universe popped out of literally nothing". Sounds like mysticism & superstition to me. In fact, even Christianity is starting to sound alot more logical in comparison. But hey, to each his own.

Subdi Visions wrote:
The argument over definitions has also been going on a long time and has thoroughly been covered by much greater minds already.


Indeed, I'm sure many intelligent atheists are pushing for a "lack of belief" definition possibly because they want to shift the burden of proof to theists and make their position the default. And let's face it, when you make an affirmative statement you actually have to back it up, so it's easier to adopt a skeptic attitude instead of a cynic attitude. The reality is that theists have their own extraordinary claims that require their own proof, regardless of what the definition of atheist is.

Here's the bottom line: virtually nobody uses the "lack of belief" definition because it's too vague. If someone said they were an atheist, you still wouldn't know if they were agnostic, undecided, believed that god doesn't exist, or never thought about it. This makes the word useless, and it's why virtually nobody outside of atheists uses this definition.

You'll usually find that the "weak atheist" or "strong atheist" terms are used only on atheist websites/blogs and the peer-edited places like wikipedia. However, if you want to appeal to authority then here's a few articles which discuss the correct definition.

Subdi Visions wrote:
theists gave us the title atheist by their common usage. We don't want to play the silly "let's play make believe" game. As far as I'm concerned you can shove the name of atheist so far up the asses of theists that they wake up out of their stupidity. Frick, frackin slippery fools.

Your chronic anger issues aside, what do you base this on? According to the 2008 American Religious Identification Survey only 5% of non-religious people labeled themselves as "atheist". It's nobody's fault that you chose a label which you feel doesn't represent you. Most likely you didn't actively look at all the options and say "hmm, atheism sounds right for me" - instead I'm assuming you read a few atheist blogs/websites & decided to follow the herd. If you don't want to use the correct etymological definition of atheism then you should adopt another position, or create your own.


Zaq
atheist
Zaq's picture
Posts: 269
Joined: 2008-12-24
User is offlineOffline
Common Fallacies

Some common fallacies I see in many arguments from design/cosmological arguments

 

Fallacy 1:  There must be a first mover

This is incorrect because all inertial reference frames are equally valid.  A universe that "begins" stationary is the same as a universe that "begins" in motion.  Also, once you have mass you have gravity, which means force, which means acceleration, which means motion.

 

Fallacy 2:  There must be either a first cause or an infinite regress

This is incorrect.  Quantum physics has inherent randomness, allowing for essentailly uncaused events.  Not everything has a cause.  Moreover, even IF there was a first cause, this in no way indicates any god-like properties, so calling the first cause God is premature.

 

Fallacy 3:  It couldn't have been chance so it must be designed

This is a false dichotomy.  Chance and design aren't the only possible explanations, so evidence against chance is not evidence for design.

 

Fallacy 4:  The universe either has a beginning or is infinitely old

This is based on a common misconception about lower bounds and minima.  For example, take the interval (0,1).  While it is true that no number in this interval is less than zero (there is a lower bound), there is still no smallest number.  Just because there is a limit to how early one can get doesn't necessarily mean there's an "earliest" time.  The universe could conceivably have no beginning (earliest moment) yet not be infinitely old.

 

Fallacy 5: God is outside the rules

The above statement would itself be a rule governing the properties of God, which makes it a contradiction.

 

Fallacy 6: Science can't explain it so we'll conclude God

This fallacy is based on a switching of the meaning for the contraction "can't" that occurs due to not specifying the time.  Science cannot currently explain it is certainly true, yet this does not necessarily mean that science is fundamentally incapable of explaining it.  Just because there's no current explanation doesn't mean science is insufficient.

 

Fallacy 7: Science is wrong so theism is right

This is the false dichotomy fallacy that Intelligent Design / Creation Science / Creationism tried to use.  Evidence against one idea is not evidence for another.  Even if one can prove that all of science is wrong, this does nothing to prove the existence or necessity for supernatural beings.

 

 

As for the OP question:  We're not sure, but the fact that we haven't figured it out yet does not mean we need to resort to supernatural causation to "explain" it.  I personally like the alternative to Fallacy 4, a sort of temporally fractal universe, but of course I have no evidence for it.  I just think it sounds cool.

Questions for Theists:
http://silverskeptic.blogspot.com/2011/03/consistent-standards.html

I'm a bit of a lurker. Every now and then I will come out of my cave with a flurry of activity. Then the Ph.D. program calls and I must fall back to the shadows.


A_Nony_Mouse
atheist
A_Nony_Mouse's picture
Posts: 2880
Joined: 2008-04-23
User is offlineOffline
.

Aedus wrote:
What are your thoughts on the origins of the universe? I guess most atheists would subscribe to some sort of cyclical multiverse theory, in which case universes & galaxies are constantly being created throughout all eternity. But is this an adequate explanation? I'm aware that most atheists will reply with "I don't know," but hopefully we can extrapolate.

Can an infinite number of universes be quantified or rationalized? Why should the "default" state of the multiverse be one that creates galaxies and then life? Even if the universe existed for perpetuity, motion is not a property of matter, so what got the multiverse moving in the first place? Theists would say this thing is God. Why can't it be God, or, what do atheists offer instead of God?

The search for gotcha questions for atheists never ceases, does it?

Is there a question so difficult to answer that even an atheist will have to admit the omni-everything Christian god exists?

Of course there are a lot of questions about the omni-everything god in the gotcha category. Could IT make a weight so heavy IT could not lift? Could it do pushups on an event horizon?

The majority of the world today gets along without an omni-everything god much less one who created the universe. In western history so did the Egyptians, Greeks, Romans, Persians and a host of -ians in Mesopotamia. Not a one of them created the universe. Belief in gods does not hinge upon giving one or more of them credit for the universe.

The simplest answer is, I don't know.

The best question to the believer is, how do you know? When the educated world dropped Adam and Eve and Abraham from the realm of real people the entire chain of authority for revelation disappeared. And if there is one thing our modern world should have taught everyone it is that the wisest of our ancestors knew less about the physical world than our children.

Attributing superior knowledge of gods to them is downright stupid.

Jews stole the land. The owners want it back. That is all anyone needs to know about Israel. That is all there is to know about Israel.

www.ussliberty.org

www.giwersworld.org/made-in-alexandria/index.html

www.giwersworld.org/00_files/zion-hit-points.phtml


theotherguy
theotherguy's picture
Posts: 294
Joined: 2007-01-07
User is offlineOffline
Aedus wrote:Exception to the

Aedus wrote:

Exception to the rules. It's really a simple concept. If we assume that a Creator exists, then we can assume that he has enough power to bring himself into being. God can be an uncaused cause - simple matter cannot.

Are you familiar with the term "special pleading?"

You are also aware that you have imposed something which you admit is much more complex than "simple matter" as the explaination for something simpler, correct?

How is it that something very complex can simply come into being, yet something very simple cannot?

Also, you can surely admit that without "being" such a creator would have no power at all, let alone the power to "bring himself into being."


latincanuck
atheist
latincanuck's picture
Posts: 2038
Joined: 2007-06-01
User is offlineOffline
Aedus wrote: The evidence is

Aedus wrote:


The evidence is creation, the conclusion is a creator.

What evidence of creation do you have that so many people, even creationist/IDers don't have, to disprove atheists that god does not exist?

Quote:

I make no claims about any of his properties, other than that he is the uncaused cause. We know that simple matter cannot be the cause of its own existence, so we must find something that can. My conclusion follows from the evidence because there are many laws of creation which defy any natural need for their existence, such as the Planck Constant (if it was higher or lower than .00000001 then life could not exist as we know it, and there is no reason for it not to be any higher or lower), the strong force (if it was a trillionth of a centimeter or less then all of the universe's hydrogen would be helium and life could not exist) and the speed of light (if it was any higher or lower life could not have formed), among other things. There is no reason for these constants to be what they are. The odds of such a universe forming are so small that they should basically be zero, which points to a creator. It's deductive reasoning.

wow the whole change one thing and everything must be different, yet we do not know exactly how it would change, because there is more than just one factor in regards to how this universe operates within the physical laws. With that said, You change one factor that could change other factors that might end up with the same result we have now, or completely different. Who knows, we only can deal with what we know, which is these are the constants we have to play with, what IF's don't count when trying to prove a creator exists.

Quote:


But then, if you subscribe to a cyclical multiverse theory which has generated an infinite # of universes throughout eternity, then the odds don't matter and such a universe would eventually be created. Here are the assumptions that go into this:

1) That the constants mentioned above are modified every time a universe is created (whether the universe is created through quantum fluctuations or collision of branes, etc. etc.) As far as I know, there is no science to support this and it's a huge assumption.

2) That this multiverse is eternal and not created. No part of science requires cyclical multiverses to be eternal, and claiming that it is eternal only presents us with more problems i.e. having to cross an infinity, which, as Kavis basically said, is a futile task.

The universe is either created or eternal. I've presented why I think that it's the former, so what is the reasoning for believing that it's the latter? It seems to me that the latter is nothing more than a horrific violation of Occam's razor, supported neither by science nor reasoning. It's like grasping at straws. No matter what most atheists say, they have taken up a position which requires them to support the latter regardless of whether or not it's actually true.

Why must it be eternal or created? what if this is the first time around? What if these is the only universe period. I mean as far as physics goes energy is eternal, doesn't mean the universe is. Second scientists have found that the universe is actually accelerating faster than previously thought, and that in a few 100 billions years it might tear itself apart. Which would take away your eternal universe concept.
 

Quote:

You misunderstand me. I'm saying that motion is not a property of matter and without motion the universe would just be a giant blob of matter making the existence of solar systems and galaxies and life as we know it impossible. It's existence is another one of the assumptions that go into the eternal universe that atheists require, despite there being no reason for it.

Yet matter can have gravity which can cause motion to other matter near it. No god required here. Why must the assumption be eternal universe again?

As well since you keep on mentioning a creator or god, what are the properties of such said being? I mean after all a creator is apparently necessary i your world view, so what are the properties of this being exactly? That doesn't require another creator and can create simple matter?


 


ClockCat
ClockCat's picture
Posts: 2265
Joined: 2009-03-26
User is offlineOffline
:3

I don't believe in a god.

I don't think there is a good likelihood of a god existing beyond people's imaginations, judging by the human desire to make spirits and gods for everything they don't understand throughout history.

 

 

Am I an atheist?

Theism is why we can't have nice things.


Aedus
Posts: 33
Joined: 2009-08-07
User is offlineOffline
Quote:Some common fallacies

Quote:
Some common fallacies I see in many arguments from design/cosmological arguments


Can you name the fallacies apart from numbering them? Simply saying they are fallacies or that I even made them does not make it so.

Zaq wrote:
Fallacy 1:  There must be a first mover

This is incorrect because all inertial reference frames are equally valid.  A universe that "begins" stationary is the same as a universe that "begins" in motion.  Also, once you have mass you have gravity, which means force, which means acceleration, which means motion.

 

Fallacy 2:  There must be either a first cause or an infinite regress

This is incorrect.  Quantum physics has inherent randomness, allowing for essentailly uncaused events.  Not everything has a cause.  Moreover, even IF there was a first cause, this in no way indicates any god-like properties, so calling the first cause God is premature.

 

Fallacy 3:  It couldn't have been chance so it must be designed

This is a false dichotomy.  Chance and design aren't the only possible explanations, so evidence against chance is not evidence for design.


These would be relevant if I were trying to convince you that God exists, but I'm not. You misunderstand my argument - I am aware that it is possible that God does not exist. I did not claim that these things have to be 100% true, I simply put them in contrast to the alternative. I was simply wondering if any atheists had a justification to this position they're compelled to support or why they think the alternatives are better. Is it inconceivable that the universe could have been created, not eternal or accidental? Which do you think is more likely and why?

 

Quote:
Fallacy 4:  The universe either has a beginning or is infinitely old

This is based on a common misconception about lower bounds and minima.  For example, take the interval (0,1).  While it is true that no number in this interval is less than zero (there is a lower bound), there is still no smallest number.  Just because there is a limit to how early one can get doesn't necessarily mean there's an "earliest" time.  The universe could conceivably have no beginning (earliest moment) yet not be infinitely old.


Very well, point taken. But now all we've done is added a 3rd option. Now the universe is either created, eternal, or accidental. "Accidental" sounding even more unlikely than the previous 2 options IMO.


Quote:
Fallacy 5: God is outside the rules

The above statement would itself be a rule governing the properties of God, which makes it a contradiction.


Ok then, change that to "The Creator is outside the rules of this universe/reality."


Quote:
Fallacy 6: Science can't explain it so we'll conclude God

This fallacy is based on a switching of the meaning for the contraction "can't" that occurse due to not specifying the time.  Science cannot currently explain it is certainly true, yet this does not necissarily mean that science is fundamentally incapable of explaining it.  Just because there's no current explanation doesn't mean science is insufficient.

Fallacy 7: Science is wrong so theism is right

This is the false dichotomy fallacy that Intelligent Design / Creation Science / Creationism tried to use.  Evidence against one idea is not evidence for another.  Even if one can prove that all of science is wrong, this does nothing to prove the existence or necessity for supernatural beings.


Aside from the fact that 6 & 7 are virtually the same thing, I'm not saying that science is wrong - I'm saying that it doesn't fit the atheist position on cosmology. Also, take supernatural beings out of the equation if you need to, it's atheist beliefs I'm putting into question here. Besides, if we define the "creator" as the instigator of the universe and the originator of biological life then it's scientifically falsifiable.

A_Nony_Mouse wrote:
Of course there are a lot of questions about the omni-everything god in the gotcha category. Could IT make a weight so heavy IT could not lift? Could it do pushups on an event horizon?


Those questions are onthological nonsense - they violate the nature of being. A rock by nature has a set weight & size. For it to be so heavy that God couldn't lift it, it would have to contradict itself. Here is a very good article on the subject: http://www.errantskeptics.org/Omnipotence.htm


latincanuck wrote:
What evidence of creation do you have that so many people, even creationist/IDers don't have, to disprove atheists that god does not exist?

wow the whole change one thing and everything must be different, yet we do not know exactly how it would change, because there is more than just one factor in regards to how this universe operates within the physical laws. With that said, You change one factor that could change other factors that might end up with the same result we have now, or completely different. Who knows, we only can deal with what we know, which is these are the constants we have to play with, what IF's don't count when trying to prove a creator exists.


First off, evidence =/= proof. You can find "evidence" for almost anything. There are many cases where asking someone to prove beyond a shadow of a doubt that something doesn't exists is ridiculous. Second off, the point of my post was to look at the alternative, which seems even more presumptuous & unreasonable.


latincanuck wrote:
Why must it be eternal or created? what if this is the first time around? What if these is the only universe period. I mean as far as physics goes energy is eternal, doesn't mean the universe is.


Then that would mean there is no explanation or reason whatsoever for how or why the universe exists - it just does. A ridiculous assertion, and even more unlikely than an eternal universe in my opinion. You might as well just accept the other two options.


latincanuck wrote:
Second scientists have found that the universe is actually accelerating faster than previously thought, and that in a few 100 billions years it might tear itself apart. Which would take away your eternal universe concept.


There are cyclic universe theories out there. In string theory, brane collisions could produce a big bang. If you assume that these branes always existed & are always colliding with each other you've got an eternal universe, even if a single universe eventually tears itself apart.


latincanuck wrote:
Yet matter can have gravity which can cause motion to other matter near it.

Motion & gravity is a property of the universe, not matter.

latincanuck wrote:
As well since you keep on mentioning a creator or god, what are the properties of such said being? I mean after all a creator is apparently necessary i your world view, so what are the properties of this being exactly? That doesn't require another creator and can create simple matter?

How would I know the properties of a supreme being & creator of the universe? All I can tell you is that I believe such a thing exists through deductive reasoning.

theotherguy wrote:
Are you familiar with the term "special pleading?"

Yes, are you? I'm assuming you somehow took up the position of atheist - a position that requires you to reject the fact that the universe is either eternal or accidental, despite the fact that the odds of such a universe existing are highly unlikely. More unlikely than the universe being created.

theotherguy wrote:
You are also aware that you have imposed something which you admit is much more complex than "simple matter" as the explaination for something simpler, correct?

How is it that something very complex can simply come into being, yet something very simple cannot? 

Also, you can surely admit that without "being" such a creator would have no power at all, let alone the power to "bring himself into being."

He doesn't have to be the one to bring himself into being - you'll also hear theists say that he is the one constant. Either way, we can assume that how the creator exists is beyond our ken. The only thing about this creator that I really claim is that he is the uncaused cause.

 


latincanuck
atheist
latincanuck's picture
Posts: 2038
Joined: 2007-06-01
User is offlineOffline
Aedus wrote: First off,

Aedus wrote:


First off, evidence =/= proof. You can find "evidence" for almost anything. There are many cases where asking someone to prove beyond a shadow of a doubt that something doesn't exists is ridiculous. Second off, the point of my post was to look at the alternative, which seems even more presumptuous & unreasonable.

First off you have to have some evidence to base you reasoning on there being a creator, otherwise it's just merely a hypothesis with nothing to back it up, meaning it's a useless concept. Second you cannot ask to prove a negative, we can only prove a positive. Third at least with some from of evidence we can see if you are just merely misinterpreting the evidence or do you have a valid concept to build upon. If you have none then that's what it means, you have nothing to back up your statement regarding god.

Quote:

Then that would mean there is no explanation or reason whatsoever for how or why the universe exists - it just does. A ridiculous assertion, and even more unlikely than an eternal universe in my opinion. You might as well just accept the other two options.

No it just means we don't have an explanation at this point of time, however it IS a possibility that this is the first and only universe at this point in time, and it might be just the only universe period.

Quote:

There are cyclic universe theories out there. In string theory, brane collisions could produce a big bang. If you assume that these branes always existed & are always colliding with each other you've got an eternal universe, even if a single universe eventually tears itself apart.

That's great but it doesn't mean they are correct, your making an assumption they are correct, however it doesn't mean they are, and so far there is no concrete evidence that string theory is correct, even more so string theory has a few major problems with it at this time. As for the cyclic universe theory again if the new evidence that the universe is accelerating and at some point it will tear is self apart then there goes the cyclic theory out as well.

Quote:

Motion & gravity is a property of the universe, not matter.

Last time I check matter is part of this universe, matter creates gravity, it's an intrinsic property of matter.


Quote:

How would I know the properties of a supreme being & creator of the universe? All I can tell you is that I believe such a thing exists through deductive reasoning.

So assign one property, the ability to create, but don't have anything else regarding your god, no properties no nothing, except that it can create...hmm so a quantum flux may have cause energy (that according to science cannot be created nor destroyed) to amass into a unstable singularity, which then releases all that energy (the BBT) that begins this universe, so you call that quantum flux god?

 


HisWillness
atheistRational VIP!
HisWillness's picture
Posts: 4100
Joined: 2008-02-21
User is offlineOffline
Aedus wrote:Before we

Aedus wrote:

Before we continue I feel that I should clarify my stance:
The evidence is creation, the conclusion is a creator. I make no claims about any of his properties, other than that he is the uncaused cause.

Yes, but apparently it's male. That would be a claim about "his" properties. Also , "he" creates. Another property. A problem arises, in that we don't exactly know what happened around what you might call "creation", so how can we know that there was a creator? In fact, is seems as though we don't know if anything really was created.

And yet, you feel confident to label this mystery "he", a gender we find in nature, among species with sexual reproduction.

Aedus wrote:
We know that simple matter cannot be the cause of its own existence, so we must find something that can.

We're not actually compelled to reach that conclusion at all, or to follow the idea that some uncaused causal force somehow creates matter. We don't yet have enough data to perfectly describe what happens at the beginning of our universe anyway, so we don't have the information we would need to jump to your conclusion at all.

Aedus wrote:
My conclusion follows from the evidence because there are many laws of creation which defy any natural need for their existence,

Your conclusion is spurious. Just because you can't find a reason for something to be a certain way, it doesn't immediately follow that any preconceived explanation is better no matter what.

Aedus wrote:
There is no reason for these constants to be what they are. The odds of such a universe forming are so small that they should basically be zero, which points to a creator. It's deductive reasoning.

Not really. Jumping from "I can't see why these constants would be this way" to "the odds of our universe forming are low" to "creator" is not an example of deductive reasoning at all. I'm not sure where you got that impression.

Saint Will: no gyration without funkstification.
fabulae! nil satis firmi video quam ob rem accipere hunc mi expediat metum. - Terence


ClockCat
ClockCat's picture
Posts: 2265
Joined: 2009-03-26
User is offlineOffline
:o

I don't understand why you need to make something magical to create an explanation for something you can't explain, rather than simply saying you don't know.

 

I mean history is nothing but full of these lessons of people again and again making gods or spirits of some kind up to comfort themselves rather than just admitting they don't know something.

 

 

What makes an older god any less valid than a newer god? What makes the god of magical burning that oversees all fire any less valid than yahweh, allah, thor, or zeus?

 

 

 

 

I don't see why you don't approach things by looking using evidence to find an answer, rather than making up an answer out of the blue and then trying to find anything you can to fit to it.

 

 

 

Making a god up to try and explain away things seems just lazy, and ignorant.

Theism is why we can't have nice things.


HisWillness
atheistRational VIP!
HisWillness's picture
Posts: 4100
Joined: 2008-02-21
User is offlineOffline
Aedus wrote:I was simply

Aedus wrote:
I was simply wondering if any atheists had a justification to this position they're compelled to support or why they think the alternatives are better. Is it inconceivable that the universe could have been created, not eternal or accidental? Which do you think is more likely and why?

I suppose it depends on how you approach ontology. For instance, if I said, "the Tooth Fairy exists," and your approach to ontology was to immediately believe me until evidence presented itself that the Tooth Fairy did not exist, then you would be operating much like a very young child, who must be trusting in order to learn. If, however, you reserved your belief for compelling arguments, then you might be more like the adult who has been tricked a few times, and might wait for evidence to see if the Tooth Fairy really exists, or if it's just something someone made up.

It is, of course, conceivable that the universe was created. Many people have conceived of that idea, and many people believe it. But is it compelling enough to convince someone who might doubt it's true? I'm not sure.

If you could be sure that any and all objects in the universe were created, then you would have certainty. We have no such knowledge, so I don't think we can reach that conclusion. Neither could you conclude that the universe is eternal or "accidental" (whatever that might mean, considering we don't even know how it started).

Aedus wrote:
Very well, point taken. But now all we've done is added a 3rd option. Now the universe is either created, eternal, or accidental. "Accidental" sounding even more unlikely than the previous 2 options IMO.

The problem with mystery is that we can actually create an infinite number of possibilities within it. For instance, my assertion that the universe was prepared by the Cosmic Burrito Line Cook is just as valid as your nondescript male "creator". I can even put a hat on him, and special shoes, and so forth. I can do that because we're talking about something neither of us can confirm. That doesn't mean we'll never be able to confirm it, we just can't right now, and so we can make up an infinity of possible scenarios based on no information whatsoever.

Aedus wrote:
Ok then, change that to "The Creator is outside the rules of this universe/reality."

You've presented yourself with a contradiction, in that case. You've made something that creates in a way that is outside the rules of our universe. There is no way for you to claim that it "creates" at all, given that we only know about creation in our universe. In fact, given that this something-or-other is outside the rules of this reality, how is it possible for you to have any knowledge of it at all?

Aedus wrote:
Besides, if we define the "creator" as the instigator of the universe and the originator of biological life then it's scientifically falsifiable.

You ask that supernatural creatures be taken out of the equation, but maintain that this creature is outside the rules of nature (that would be "supernatural" ). I think maybe you should clarify.

Aedus wrote:
Those questions are onthological nonsense - they violate the nature of being.

You'll notice that any questions about supernatural creatures violate the nature of being.

Aedus wrote:
what IF's don't count when trying to prove a creator exists.

Can I quote you on that? You're asking us to entertain the possibility of a creator, so ...

Aedus wrote:
Then that would mean there is no explanation or reason whatsoever for how or why the universe exists - it just does. A ridiculous assertion, and even more unlikely than an eternal universe in my opinion. You might as well just accept the other two options.

Hold on -- in the absence of knowledge, what assertions are ridiculous? We don't know the cause of our universe. That's clear. So whatever you want to guess is there doesn't have any evidence we can apply to it (or against it).

Motion & gravity is a property of the universe, not matter.

Aedus wrote:
I'm assuming you somehow took up the position of atheist - a position that requires you to reject the fact that the universe is either eternal or accidental, despite the fact that the odds of such a universe existing are highly unlikely. More unlikely than the universe being created.

That would be a strange set of assumptions. Having an opinion or belief on vast areas of human understanding isn't so much "atheism" as it is "bar conversations". If our universe is extremely unlikely to exist AND exists, then where does your argument go from there? Jumping to "therefore creator" is a strong non sequitur. It could be that our universe really is unlikely to exist, just as it's extremely unlikely that a person might win the lottery, but some people do.

Aedus wrote:
Either way, we can assume that how the creator exists is beyond our ken. The only thing about this creator that I really claim is that he is the uncaused cause.

That's a contradiction itself. This creator is beyond our ken, but somehow you know something about it.

Saint Will: no gyration without funkstification.
fabulae! nil satis firmi video quam ob rem accipere hunc mi expediat metum. - Terence


theotherguy
theotherguy's picture
Posts: 294
Joined: 2007-01-07
User is offlineOffline
Aedus wrote:theotherguy

Aedus wrote:

theotherguy wrote:
Are you familiar with the term "special pleading?"

Yes, are you? I'm assuming you somehow took up the position of atheist - a position that requires you to reject the fact that the universe is either eternal or accidental, despite the fact that the odds of such a universe existing are highly unlikely. More unlikely than the universe being created.

Let's look at this statement that you just made, "the odds of such a universe existing are highly unlikely." How is this so? To make a statement of probability, you need an event space of many possibilities, and you must count the events that have a universe in them and divide that event space by the total number of events. Now tell me, how did you make this calculation? How did you come to the conclusion that it is more likely that the universe was "created" than was "accidental?" Usually, when people are calculating probabilites of events which cannot be mathematically calculated with ease, they perform an experiment, or a study, to determine the probabilities of events. Unfortunately, we can only observe one universe -- that is, we can only perform one experiment, so we cannot assign any probabilities to its existence which are not pure speculation.

You have created a false dilemma by saying the universe must be "eternal or accidental" or in your case, created. First, let us define a cause. We say that an event K "causes" event E when K precedes E, and when E could not have occurred without K occurring, (or, more succinctly,K=>E).  It follows that in order for K to cause E, K must exist earlier in time than E--otherwise we cannot say which causes the other.

Now, as you have so kindly stated, everything we have observed in the universe must have a cause. Why is this so? Causality is a fundamental feature of the universe. It is a consequence of time, and entropy. We observe that time is fundamentally entwined with space. Indeed, Einstein showed that they were the same thing. What is the universe? The universe is a set of entities. It is time, space, energy and matter.

It follows that, since time is a subset of the universe,  in order for something to cause another thing, it must exist within the universe. It follows from this that nothing could have caused the universe, because without the universe, time is undefined, and hence causality is undefined.

 

 

Aedus wrote:

theotherguy wrote:
You are also aware that you have imposed something which you admit is much more complex than "simple matter" as the explanation for something simpler, correct?

How is it that something very complex can simply come into being, yet something very simple cannot? 

Also, you can surely admit that without "being" such a creator would have no power at all, let alone the power to "bring himself into being."

He doesn't have to be the one to bring himself into being - you'll also hear theists say that he is the one constant. Either way, we can assume that how the creator exists is beyond our ken. The only thing about this creator that I really claim is that he is the uncaused cause.

Please explain why the universe needs a cause and your deity does not.


Aedus
Posts: 33
Joined: 2009-08-07
User is offlineOffline
latincanuck wrote:First off

latincanuck wrote:
First off you have to have some evidence to base you reasoning on there being a creator, otherwise it's just merely a hypothesis with nothing to back it up, meaning it's a useless concept.

Kind of like atheism then? Read the part where I clarify my position. Again, proof =/= evidence. Probability and even heresay still count as evidence. If you disagree then feel free to go up to any court of law & tell them otherwise.

 

latincanuck wrote:
Second you cannot ask to prove a negative, we can only prove a positive.

No. Burden of proof depends on whether something is an affirmative statement, not on whether it's a negative statement. If I told you that "19 is not a positive number" then is the burden of proof on you and not me? No - that would be ridiculous.

 

latincanuck wrote:
Third at least with some from of evidence we can see if you are just merely misinterpreting the evidence or do you have a valid concept to build upon. If you have none then that's what it means, you have nothing to back up your statement regarding god.

I have the probability that is more likely for him to exist than not. There's a difference between that and trying to prove his existence. And it is usually atheists who misrepresent science with their claims such as "science proves god does not exist". But hopefully I won't have to get into that?

 

Quote:
No it just means we don't have an explanation at this point of time, however it IS a possibility that this is the first and only universe at this point in time, and it might be just the only universe period.

Just looks like a bunch of unfounded assumptions to me. If you think something else caused the big bang then feel free to talk about it & see if the basic logic even makes sense. I'm not asking about the details.

 

Quote:
That's great but it doesn't mean they are correct, your making an assumption they are correct, however it doesn't mean they are, and so far there is no concrete evidence that string theory is correct, even more so string theory has a few major problems with it at this time. As for the cyclic universe theory again if the new evidence that the universe is accelerating and at some point it will tear is self apart then there goes the cyclic theory out as well.

"Universe" is sometimes used in place of "multiverse."

 

Quote:
Last time I check matter is part of this universe, matter creates gravity, it's an intrinsic property of matter.

No, it's not. Matter bends space-time which causes objects close to each other to come together. Space-time is a property of the universe, not matter. As is the quantum vacuum & all the other assumptions that your atheist universe requries to exist without a creator.

 

Quote:
So assign one property, the ability to create, but don't have anything else regarding your god, no properties no nothing, except that it can create...hmm so a quantum flux may have cause energy (that according to science cannot be created nor destroyed) to amass into a unstable singularity, which then releases all that energy (the BBT) that begins this universe, so you call that quantum flux god?

No, that's not how I defined the creator. If we need to nitpick then from now on let's refer to deism's version of the creator.


 

HisWillness wrote:
Yes, but apparently it's male. That would be a claim about "his" properties. Also , "he" creates. Another property.

That's disingenuous. I think we both know I refer to him as a "he" out of habit.

 

HisWillness wrote:
A problem arises, in that we don't exactly know what happened around what you might call "creation", so how can we know that there was a creator? In fact, is seems as though we don't know if anything really was created.

We can't know for sure - you're not reading my posts.

 

Quote:
We're not actually compelled to reach that conclusion at all, or to follow the idea that some uncaused causal force somehow creates matter. We don't yet have enough data to perfectly describe what happens at the beginning of our universe anyway, so we don't have the information we would need to jump to your conclusion at all.

Point taken. But as I asked before, why is it more likely that the universe is eternal or accidental than created? I've given you reasons why I think it's created, not the other two. What is your reasoning? If you have none then we can agree that it is more likely that the universe was created than it is anything else, based on our current knowledge.

Quote:
Your conclusion is spurious. Just because you can't find a reason for something to be a certain way, it doesn't immediately follow that any preconceived explanation is better no matter what.

Again, I never said it was - I'm arguing parsimony.

 

Quote:
Not really. Jumping from "I can't see why these constants would be this way" to "the odds of our universe forming are low" to "creator" is not an example of deductive reasoning at all. I'm not sure where you got that impression.

The odds of our universe being created with all the constants fine-tuned has been calculated to be 10000 to power of 124. Do you disagree that this makes the idea that our universe just popped into existence on its own unlikely?

 

theotherguy wrote:
Let's look at this statement that you just made, "the odds of such a universe existing are highly unlikely." How is this so? To make a statement of probability, you need an event space of many possibilities, and you must count the events that have a universe in them and divide that event space by the total number of events. Now tell me, how did you make this calculation? How did you come to the conclusion that it is more likely that the universe was "created" than was "accidental?" Usually, when people are calculating probabilites of events which cannot be mathematically calculated with ease, they perform an experiment, or a study, to determine the probabilities of events. Unfortunately, we can only observe one universe -- that is, we can only perform one experiment, so we cannot assign any probabilities to its existence which are not pure speculation.

 

It's no more "pure speculation" than any speculative science is - I'm using observations about the natural world to make inferences. Donald Page of the Princeton Institute of Advanced Study has calculated the odds of our universe forming without God putting in the necessary constants is about 10,000,000 to the power 124. You can take this up with him if you disagree. And really, it's not so much about probability as it is about parsimony - my position makes way fewer assumptions.

 

theotherguy wrote:
You have created a false dilemma by saying the universe must be "eternal or accidental" or in your case, created. First, let us define a cause. We say that an event K "causes" event E when K precedes E, and when E could not have occurred without K occurring, (or, more succinctly,K=>E).  It follows that in order for K to cause E, K must exist earlier in time than E--otherwise we cannot say which causes the other.

 

Now, as you have so kindly stated, everything we have observed in the universe must have a cause. Why is this so? Causality is a fundamental feature of the universe. It is a consequence of time, and entropy. We observe that time is fundamentally entwined with space. Indeed, Einstein showed that they were the same thing. What is the universe? The universe is a set of entities. It is time, space, energy and matter.

 

It follows that, since time is a subset of the universe,  in order for something to cause another thing, it must exist within the universe. It follows from this that nothing could have caused the universe, because without the universe, time is undefined, and hence causality is undefined.

 

In other words, you subscribe to the "a fireball appeared in the sky for no apparent reason & caused the big bang" idea? Seriously, what is your alternative? Also, your example depends entirely on causality being attributed to time. Ontological causality does not suggest the temporal relation of before and after. And how do you know that causality would be undefined? You have no idea whether causality is invalid outside of our universe. This is just a bunch of assumptions.

 

HisWillness wrote:
The problem with mystery is that we can actually create an infinite number of possibilities within it. For instance, my assertion that the universe was prepared by the Cosmic Burrito Line Cook is just as valid as your nondescript male "creator". I can even put a hat on him, and special shoes, and so forth. I can do that because we're talking about something neither of us can confirm. That doesn't mean we'll never be able to confirm it, we just can't right now, and so we can make up an infinity of possible scenarios based on no information whatsoever.

As long as we're clear that my creator is much more likely to exist than yours since mine makes the fewest assumptions.

 

Quote:
You've presented yourself with a contradiction, in that case. You've made something that creates in a way that is outside the rules of our universe. There is no way for you to claim that it "creates" at all, given that we only know about creation in our universe.

If it's semantics you want to argue then you can feel free to suggest a better term than "creates." Nobody can see electrons but we can postulate that they exist and we can say that they have a negative charge despite the fact that "negative" is just a name we've ascribed to it so that we ourselves can better understand it.

Quote:
You ask that supernatural creatures be taken out of the equation, but maintain that this creature is outside the rules of nature (that would be "supernatural" ). I think maybe you should clarify.

Yep you got me. Sticking out tongue

Supernatural as in made-up/fictional. And the creator does not have to be supernatural - remember I'm making as few assumptions about "him" as possible.

Quote:
Hold on -- in the absence of knowledge, what assertions are ridiculous? We don't know the cause of our universe. That's clear. So whatever you want to guess is there doesn't have any evidence we can apply to it (or against it).

We know enough to make educated guesses. It may not be testable now, but it could potentially be testable later.

 

Quote:
That would be a strange set of assumptions. Having an opinion or belief on vast areas of human understanding isn't so much "atheism" as it is "bar conversations". If our universe is extremely unlikely to exist AND exists, then where does your argument go from there? Jumping to "therefore creator" is a strong non sequitur. It could be that our universe really is unlikely to exist, just as it's extremely unlikely that a person might win the lottery, but some people do.

Neither you nor anybody in this topic has shown why jumping to "therefore creator" is a non-sequitur, or why it's any more unlikely than the alternatives.

 

Quote:
That's a contradiction itself. This creator is beyond our ken, but somehow you know something about it.

And you know what electrons could look like despite the fact that you've never seen them. So what?

 

ClockCat wrote:
I don't understand why you need to make something magical to create an explanation for something you can't explain, rather than simply saying you don't know.

No need for it to be magical. It's deductive reasoning.

 

ClockCat wrote:
I mean history is nothing but full of these lessons of people again and again making gods or spirits of some kind up to comfort themselves rather than just admitting they don't know something.

 

What makes an older god any less valid than a newer god? What makes the god of magical burning that oversees all fire any less valid than yahweh, allah, thor, or zeus?

Making a god up to try and explain away things seems just lazy, and ignorant.

What are you talking about? I feel like everything I've been typing this whole time has been lost on you because you either aren't reading it or don't understand it. Nobody is talking about different versions of gods or history. Those things have nothing to do with this thread.

ClockCat wrote:
I don't see why you don't approach things by looking using evidence to find an answer, rather than making up an answer out of the blue and then trying to find anything you can to fit to it.

 

I don't see why people can't read my actual position before simply claiming that I have no evidence.

 


MichaelMcF
Science Freak
MichaelMcF's picture
Posts: 525
Joined: 2008-01-22
User is offlineOffline
Aedus wrote:The evidence is

Aedus wrote:

The evidence is creation, the conclusion is a creator.

 

The evidence is creation?  The word in itself assumes a creation event, so how can you then conclude a creator when you've already presumed one?

 

Aedus wrote:

I make no claims about any of his properties, other than that he is the uncaused cause. We know that simple matter cannot be the cause of its own existence, so we must find something that can.

 

Why?  Why is there an assumption that things were created or had to have come from nothing?  What if the universe is and always will be?  What if nothing was ever created?

 

Aedus wrote:

My conclusion follows from the evidence because there are many laws of creation which defy any natural need for their existence, such as the Planck Constant (if it was higher or lower than .00000001 then life could not exist as we know it, and there is no reason for it not to be any higher or lower)

 

The planck constant is 6.63x10-34 J s.  What does your above statement even mean?

 

Aedus wrote:

the strong force (if it was a trillionth of a centimeter or less then all of the universe's hydrogen would be helium and life could not exist) and the speed of light (if it was any higher or lower life could not have formed), among other things. There is no reason for these constants to be what they are.

 

Why does there need to be a reason for these constants to have the values?  This whole argument is dependent on the idea that life is somehow the ultimate goal of the universe.  The values are what they are.  The fact that the universe can support life could be taken as a neat little side effect - and something we are only capable of commenting on because we exist.  I think Douglas Adams put it best:

"Imagine a puddle waking up one morning and thinking, 'This is an interesting world I find myself in, an interesting hole I find myself in, fits me rather neatly, doesn't it? In fact it fits me staggeringly well, must have been made to have me in it!' This is such a powerful idea that as the sun rises in the sky and the air heats up and as, gradually, the puddle gets smaller and smaller, it's still frantically hanging on to the notion that everything's going to be alright, because this world was meant to have him in it, was built to have him in it; so the moment he disappears catches him rather by surprise. I think this may be something we need to be on the watch out for."

 

Aedus wrote:

The odds of such a universe forming are so small that they should basically be zero, which points to a creator. It's deductive reasoning.

Really?  That's quite astounding.  Could you please teach me some of the maths involved here?  How do you calculate the odds of a universe forming?  What variables to you have to take account of to calculate the odds of a specific universe forming (assuming of course that more than one type of universe can form).

I'm fascinated at the level of knowledge you must have to be able to make such a statement.



Adeus wrote:

But then, if you subscribe to a cyclical multiverse theory which has generated an infinite # of universes throughout eternity, then the odds don't matter and such a universe would eventually be created...

But as others have already said, most atheists don't necessarily follow a multiverse theory.  I don't know where you got this idea from.



Aedus wrote:

The universe is either created or eternal. I've presented why I think that it's the former, so what is the reasoning for believing that it's the latter? It seems to me that the latter is nothing more than a horrific violation of Occam's razor, supported neither by science nor reasoning. It's like grasping at straws. No matter what most atheists say, they have taken up a position which requires them to support the latter regardless of whether or not it's actually true.

 

But most people will not state that the universe is eternal.  I think what you'll find is that many atheists, in response to the statement that a god is eternal, will ask the question "why can't this be true for the universe?".  A more honest scientist will say "we don't know" and leave it at that.

If you're going to invoke Occam's razor then please explain why the addition of an unspecified creator - who gets special pleading for an eternal existence - does not introduce more questions and assumptions than saying "we don't know".

 

Aedus wrote:

I can accept things such as particle-wave duality once they already exist - they are properties of this universe. This and other strange laws of nature defy any natural or sufficient reasons for their existence

 

Again, why do things need a reason for their existence?  Where are you getting this idea from?  Do things need reason?  Why can't they just exist?

 

Aedus wrote:

No branch of science deals with whether the universe is eternal or not, and from what I've seen I'm not compelled to believe that it's the former. Also, science for all its accomplishments still has some limitations in the here & now. What it can't directly measure it ignores.

Mostly because no branch of science is currently capable of answering that question.  Science does not ignore what it can't measure, nor do scientists.  We try to find ways of measuring them.


Aedus wrote:

eternal universe that atheists require, despite there being no reason for it.

Atheists don't require an eternal universe.  Thanks for playing though.

 

Aedus wrote:

Interesting. Do you consider all deductive inferences to be "make believe" as well? If so then I guess that yes, my god is make believe. Just like the idea of extraterrestrial life - we haven't seen it and have no proof for it, therefore it's make believe.

No.  We haven't seen extraterrestrial life but we do have evidence of life and we have evidence that the building blocks of life can exist elsewhere in the universe.  This makes alien life a reasonable deduction.  We have no evidence for universe creators - having never witnessed a universe being created - and thus anything said about such a being is simply an assumption not deduction.

 

Aedus wrote:

Exception to the rules. It's really a simple concept. If we assume that a Creator exists, then we can assume that he has enough power to bring himself into being. God can be an uncaused cause - simple matter cannot.

Special Pleading.

 

Aedus wrote:

I find it better than "the universe just always existed - end of story"

Why?  It's the same argument you make for God - except you special plead (as above) to say that this can only be true of him/it.

 

Forget Jesus, the stars died so that you could be here
- Lawrence Krauss


MichaelMcF
Science Freak
MichaelMcF's picture
Posts: 525
Joined: 2008-01-22
User is offlineOffline
Aedus wrote:The odds of our

Aedus wrote:

The odds of our universe being created with all the constants fine-tuned has been calculated to be 10000 to power of 124. Do you disagree that this makes the idea that our universe just popped into existence on its own unlikely?

 

As a follow-up:  Where did you get this number?  You're not the first to claim that the odds of the universe forming are 1 in 10124 (or 1 in 10127 as you're claiming here).  Where did this come from?

 

By the way, before you get carried away with big numbers, if I shuffle a pack of cards what are the odds of me getting the 52 cards in any particular order?  It's 1 in 8x1067.   And that's something that can happen.  Funny that.

Forget Jesus, the stars died so that you could be here
- Lawrence Krauss


HisWillness
atheistRational VIP!
HisWillness's picture
Posts: 4100
Joined: 2008-02-21
User is offlineOffline
Aedus wrote:HisWillness

Aedus wrote:

HisWillness wrote:
Yes, but apparently it's male. That would be a claim about "his" properties. Also , "he" creates. Another property.

That's disingenuous. I think we both know I refer to him as a "he" out of habit.

Okay, fine. "It" is good. It creates universes.

Aedus wrote:
HisWillness wrote:
A problem arises, in that we don't exactly know what happened around what you might call "creation", so how can we know that there was a creator? In fact, is seems as though we don't know if anything really was created.

We can't know for sure - you're not reading my posts.

I am. You're reaching conclusions rather than saying, "it could be this or that". I would agree that it's possible that a creator could have created the universe. Barring no evidence for that, any number of other scenarios are just as likely.

Aedus wrote:
Point taken. But as I asked before, why is it more likely that the universe is eternal or accidental than created? I've given you reasons why I think it's created, not the other two. What is your reasoning? If you have none then we can agree that it is more likely that the universe was created than it is anything else, based on our current knowledge.

Again, those are not the only possibilities. What about a creator that accidentally created the universe? We could just as easily be a sneeze. Are you assuming intentionality?

Aedus wrote:
The odds of our universe being created with all the constants fine-tuned has been calculated to be 10000 to power of 124.

There is only one known universe. How would anyone have any numbers to put into a calculation of how probable a universe is? I'm afraid that calculation is complete nonsense.

Aedus wrote:
Do you disagree that this makes the idea that our universe just popped into existence on its own unlikely?

Who said it popped into existence? So far, you've created this merry-go-round of strawmen, false dichotomies and non sequiturs, and now you want me to answer a question about likelihood regarding a suspect calculation. You'll excuse me if I'm not taking you seriously at this point.

Aedus wrote:
It's no more "pure speculation" than any speculative science is - I'm using observations about the natural world to make inferences. Donald Page of the Princeton Institute of Advanced Study has calculated the odds of our universe forming without God putting in the necessary constants is about 10,000,000 to the power 124. You can take this up with him if you disagree. And really, it's not so much about probability as it is about parsimony - my position makes way fewer assumptions.

Right. That something with intentionality created the universe with specific constants. No ... no assumptions there.

Aedus wrote:
HisWillness wrote:
The problem with mystery is that we can actually create an infinite number of possibilities within it. For instance, my assertion that the universe was prepared by the Cosmic Burrito Line Cook is just as valid as your nondescript male "creator". I can even put a hat on him, and special shoes, and so forth. I can do that because we're talking about something neither of us can confirm. That doesn't mean we'll never be able to confirm it, we just can't right now, and so we can make up an infinity of possible scenarios based on no information whatsoever.

As long as we're clear that my creator is much more likely to exist than yours since mine makes the fewest assumptions.

You're proposing an intentional creator of the universe. Intentionality is an entire host of assumptions wrapped into one concept, and I put shoes on the guy and I'm adding something?

Aedus wrote:
If it's semantics you want to argue then you can feel free to suggest a better term than "creates." Nobody can see electrons but we can postulate that they exist and we can say that they have a negative charge despite the fact that "negative" is just a name we've ascribed to it so that we ourselves can better understand it.

Note that electrons can be measured. That would be empirical evidence.

Aedus wrote:
And the creator does not have to be supernatural - remember I'm making as few assumptions about "him" as possible.

I see. So the creator is natural, then. A universe creating space alien, perhaps?

Aedus wrote:
Quote:
Hold on -- in the absence of knowledge, what assertions are ridiculous? We don't know the cause of our universe. That's clear. So whatever you want to guess is there doesn't have any evidence we can apply to it (or against it).

We know enough to make educated guesses. It may not be testable now, but it could potentially be testable later.

What could we test? You've been back and forth about whether or not this thing we're testing for is natural or not. You don't even know how to apply attributes to it! There's no way to test for something when you don't know what it is. If it's natural, or at least part of it is natural, then that part abides by physical rules. As you say, if those constants were violated at some level, then life wouldn't be the way it is.

Aedus wrote:
Neither you nor anybody in this topic has shown why jumping to "therefore creator" is a non-sequitur, or why it's any more unlikely than the alternatives.

Oh, I agree that "therefore creator" is just as unlikely as anything else we could make up, but you seem to be under the assumption that a creator making the universe by accident is less likely than a creator making the universe on purpose. I'm not sure where you get that.

Aedus wrote:
Quote:
That's a contradiction itself. This creator is beyond our ken, but somehow you know something about it.

And you know what electrons could look like despite the fact that you've never seen them. So what?

You can measure electrons, and so can I. We have indirect evidence in the form of a volt meter. If you have access to a university, you can even visit their electron microsope, and you have evidence of the electrons by calculating the path required to give you the resulting image.

We have nothing for a creator. Electron evidence, check; creator evidence, no.

Aedus wrote:
I don't see why people can't read my actual position before simply claiming that I have no evidence.

It's not evidence if it doesn't point to the thing you're trying to show. If you have a hypothesis, then let's hear it. The thing about a hypothesis, though, is that it has to be specific, and it has to give us some kind of prediction. Like, "if the universe was created by a creator, then such-and-such should be true."

In your case, you've said, "the constants of the universe are a certain way, and the most likely way that could happen is if there was a creator" You even hint at an intentional creator.

However, obviously, that's not the case. There's know way for you to know how probable a one-time thing is, or how necessary it is to have an intentional creator for anything. You're simply labelling mystery, and again, you seem to want to take this one explanation as likely, ignoring the myriad others which are just as likely (as far as we know).

Saint Will: no gyration without funkstification.
fabulae! nil satis firmi video quam ob rem accipere hunc mi expediat metum. - Terence


Aedus
Posts: 33
Joined: 2009-08-07
User is offlineOffline
Quote:Really?  That's quite

Quote:
Really?  That's quite astounding.  Could you please teach me some of the maths involved here?  How do you calculate the odds of a universe forming?  What variables to you have to take account of to calculate the odds of a specific universe forming (assuming of course that more than one type of universe can form).

I'm fascinated at the level of knowledge you must have to be able to make such a statement.

Quote:
As a follow-up:  Where did you get this number?  You're not the first to claim that the odds of the universe forming are 1 in 10124 (or 1 in 10127 as you're claiming here).  Where did this come from?

It's from "Donald Page of the Princeton Institute of Advanced Study," who apparently doesn't study there anymore. Here is a page w/ a collection of references from atheists/agnostics which shows that this number is in fact high, or at the least that there are many factors which would make such a number high: http://whywontgodhealamputees.com/forum/index.php?topic=20365.msg493734#msg493734

I don't know what to say if you doubt that the #s are very high - this fact can be found on many cosmology pages.

MichaelMcF wrote:
The evidence is creation?  The word in itself assumes a creation event, so how can you then conclude a creator when you've already presumed one?

The planck constant is 6.63x10-34 J s.  What does your above statement even mean?

Well this is obviously pure nitpickery. Why does nobody address my main point? For your second statement I meant "by" instead of "than". Hopefully I don't have to explain what I meant by the first statement, eh? Bare with me here - I'm talking with like 10 of you guys.

 

MichaelMcF wrote:
Why does there need to be a reason for these constants to have the values?

Uhm, so that we can live? This article explains it pretty well: http://islam.thetruecall.com/modules.php?name=News&file=article&sid=362

 

Quote:
But as others have already said, most atheists don't necessarily follow a multiverse theory.  I don't know where you got this idea from.

Then feel free to defend however you think the universe came to be. Is it created, eternal or accidental? And why?

Quote:
If you're going to invoke Occam's razor then please explain why the addition of an unspecified creator - who gets special pleading for an eternal existence - does not introduce more questions and assumptions than saying "we don't know".

An agnostic attitude is perfectly fine to take.

Quote:
No.  We haven't seen extraterrestrial life but we do have evidence of life and we have evidence that the building blocks of life can exist elsewhere in the universe.  This makes alien life a reasonable deduction.  We have no evidence for universe creators - having never witnessed a universe being created - and thus anything said about such a being is simply an assumption not deduction.

So what? We also have evidence of a McDonalds - does that mean you'll find them on other planets? Indeed, going by atheist logic I'll go ahead & say that unless a borg cube holds orbit and destroys the white house then it's unreasonable & superstitious to think that ET life exists. What, am I supposed to believe that we have little green men running around on other planets just because there are hundreds of movies on the subject?

Also, out of curiosity, can you point me to this evidence that the building blocks of life can exist elsewhere in the universe?

Quote:
Why?  It's the same argument you make for God - except you special plead (as above) to say that this can only be true of him/it.

And why not? If anything can be an uncaused cause then wouldn't it be god? You have better ideas? Also, I suggest you familiarize yourself with my actual argument better before you label it as "special pleading". I've seriously explained my stance like a dozen times in this thread.

HisWillness wrote:
I am. You're reaching conclusions rather than saying, "it could be this or that". I would agree that it's possible that a creator could have created the universe.

No, I am in fact saying that it could be this or that. In my personal opinion one scenario is more likely than another.

 

Quote:
Again, those are not the only possibilities. What about a creator that accidentally created the universe? We could just as easily be a sneeze. Are you assuming intentionality?

That the universe is either created, eternal, or accidental? A creator that accidentally created it would fall into the 1st category. Intentionality is irrelevant.

 

Quote:
There is only one known universe. How would anyone have any numbers to put into a calculation of how probable a universe is? I'm afraid that calculation is complete nonsense.

Quote:
However, obviously, that's not the case. There's know way for you to know how probable a one-time thing is

Click on the links I provided at the beginning of this post to see why this assertion is wrong. Scientists can in fact extrapolate the range of values for these constants...

 

Quote:
Who said it popped into existence? So far, you've created this merry-go-round of strawmen, false dichotomies and non sequiturs, and now you want me to answer a question about likelihood regarding a suspect calculation. You'll excuse me if I'm not taking you seriously at this point.

This merry-go-round is simply a delusion that arises from nitpicking bits from my individual posts and ignoring the main argument that I'm presenting.

Quote:
Right. That something with intentionality created the universe with specific constants. No ... no assumptions there.

There are assumptions, but less than the alternatives. The only assumptions are that the universe was created by something intelligent. This is the last time I will repeat this. Whether or not you get it from this point forward is none of my concern.

Quote:
Note that electrons can be measured. That would be empirical evidence.

Note that electrons were predicted before they could be measured.

Quote:
In your case, you've said, "the constants of the universe are a certain way, and the most likely way that could happen is if there was a creator"

Any other explanation than a creator would have the numbers be picked randomly, and do you have any idea how large 10,000 to the 124 is? Scientists already agree that there are variables that are in fact adjusted in our favor despite the fact that there is no reason for them being so. Sure, in the presence of such overwhelming evidence I am forced to draw conclusions. No doubt somebody will respond with something like "just because the odds are high does not mean it's impossible, especially since we exist," but I can always hope someone will respond to my actual argument!


Vastet
atheistBloggerSuperfan
Vastet's picture
Posts: 13234
Joined: 2006-12-25
User is offlineOffline
"No branch of science deals

"No branch of science deals with whether the universe is eternal or not, and from what I've seen I'm not compelled to believe that it's the former."

Your biggest problem is being compelled to believe ANYTHING without proof.

Enlightened Atheist, Gaming God.


Tapey
atheist
Tapey's picture
Posts: 1478
Joined: 2009-01-23
User is offlineOffline
Well I don't see anyproof

Well I don't see anyproof here, then again I know nothing about the subject. No matter the odds it can happen. Let me give you an example.  I garentee you I could produce a guy who wins 1 thousand tosses of a coin in a row. What are the chances of that? yet it is simple to achteve, in fact I could do it with 1 million coin tosses in a row. All you need is enough people versing each other where the winners move on, eventually if you have enough people at the start you will get someone who has won 1000 games in a row. Is this even relevant i have no clue, i claim no knowledge on the subject. Perhaps a universe was bound to appear like there was bound to be someone who won 1000 tosses of the dice in a row. Just because life as we know it exists doesn't mean under differant conditions a difefrant "form" of live couldn't exist. once again I claim no knowledge.

 

Btw im not sure but your arguement is about possibility right? Then wouldn't it be better to say there probably was a creator.

 

P.S. its late and brain functions have already shut down so forgive me if i read your stuff wrong

Whatever goes upon two legs is an enemy.
Whatever goes upon four legs, or has wings, is a friend.
No animal shall wear clothes.
No animal shall sleep in a bed.
No animal shall drink alcohol.
No animal shall kill any other animal.
All animals are equal.


butterbattle
ModeratorSuperfan
butterbattle's picture
Posts: 3945
Joined: 2008-09-12
User is offlineOffline
Aedus wrote:but I can always

Aedus wrote:
but I can always hope someone will respond to my actual argument!

What's your argument again?

 

Our revels now are ended. These our actors, | As I foretold you, were all spirits, and | Are melted into air, into thin air; | And, like the baseless fabric of this vision, | The cloud-capped towers, the gorgeous palaces, | The solemn temples, the great globe itself, - Yea, all which it inherit, shall dissolve, | And, like this insubstantial pageant faded, | Leave not a rack behind. We are such stuff | As dreams are made on, and our little life | Is rounded with a sleep. - Shakespeare


MichaelMcF
Science Freak
MichaelMcF's picture
Posts: 525
Joined: 2008-01-22
User is offlineOffline
Aedus wrote:It's from

Aedus wrote:

It's from "Donald Page of the Princeton Institute of Advanced Study," who apparently doesn't study there anymore. Here is a page w/ a collection of references from atheists/agnostics which shows that this number is in fact high, or at the least that there are many factors which would make such a number high: http://whywontgodhealamputees.com/forum/index.php?topic=20365.msg493734#msg493734

I don't know what to say if you doubt that the #s are very high - this fact can be found on many cosmology pages.

 

Ah, I've had a look round and it seems the number comes from Roger Penrose.  The problem with this (if it is true - still trying to get my hands on the original material) is that Penrose's number is that it is tied to the probability of the universe occupying the state it's in after a big bang event.  This has nothing to do with creation or "origin" so you can stop throwing that number around.

 

Aedus wrote:

Well this is obviously pure nitpickery. Why does nobody address my main point?

No, this is not nitpickery.  You are assuming creation to come to the conclusion of creation.  It's circular logic.  Your main point also assumes that the default position is a multiverse theory when that is blatantly untrue.  You ask what atheists have to offer instead of God - I'm telling you "I don't know" is the only answer that's sensible.

 

Aedus wrote:

For your second statement I meant "by" instead of "than". Hopefully I don't have to explain what I meant by the first statement, eh? Bare with me here - I'm talking with like 10 of you guys.

Yeah... but I put Planck's number down for a reason.  You said "if it was higher or lower than .00000001 then life could not exist as we know it, and there is no reason for it not to be any higher or lower".  So I'll make the linguistic correction you're suggesting and now you say "if it was higher or lower by .00000001 then life could not exist as we know it, and there is no reason for it not to be any higher or lower"

 

The Planck constant is 6.63x10-34.  You're arguing that if the number was altered by 1x10-8 then things would be different.  This number is 1x1025 times larger than Planck's constant.  That's quite a large difference you're asking for there.  That's the difference between the weight of a bag of sugar and almost 2 times the weight of the earth.  On that kind of scale I'm not surprised that changing Planck's number would have strange effects!

 

Aedus wrote:

MichaelMcF wrote:
Why does there need to be a reason for these constants to have the values?

Uhm, so that we can live? This article explains it pretty well: http://islam.thetruecall.com/modules.php?name=News&file=article&sid=362

But you're getting mixed up here.   These numbers allow life to exist, life exists, therefore we see these numbers.  That doesn't mean that these numbers were tuned to allow life - that assumes purpose were none exists.

 

Aedus wrote:

Then feel free to defend however you think the universe came to be. Is it created, eternal or accidental? And why?

 

I do not know, and I think we can never know.  I also don't like the use of accidental.  Accidental often equates to "mistake" and can be misleading.

 

Aedus wrote:

So what? We also have evidence of a McDonalds - does that mean you'll find them on other planets? Indeed, going by atheist logic I'll go ahead & say that unless a borg cube holds orbit and destroys the white house then it's unreasonable & superstitious to think that ET life exists. What, am I supposed to believe that we have little green men running around on other planets just because there are hundreds of movies on the subject?

This is ridiculous and you know it.  McDonalds has a creator (as a company, a franchise and a building) that we can tie down as distinctly human, thus there is no reason to assume that we'll find McDonalds on other planets.

Your borg cube theory also only works if it is assumed that alien life must be borg like entities intent on destroying the white house.  If we're just talking about the logical possibility of life having evolved elsewhere then you're argument falls apart as it holds absolutley no connection to the base discussion.

And no-one suggested you use movies as a reference point (or that aliens would be little and green) where did you pick this particular straw man from?

 

Aedus wrote:

Also, out of curiosity, can you point me to this evidence that the building blocks of life can exist elsewhere in the universe?

Certainly

 

Aedus wrote:

And why not? If anything can be an uncaused cause then wouldn't it be god? You have better ideas? Also, I suggest you familiarize yourself with my actual argument better before you label it as "special pleading". I've seriously explained my stance like a dozen times in this thread.

But it is special pleading.  You assume that the universe must have a cause.  You then say something must have caused it and this uncaused cause is god.  I ask you why the universe can't be an uncaused caused.  To say that god can be an uncaused cause but the universe can't is special pleading.

 

 

Forget Jesus, the stars died so that you could be here
- Lawrence Krauss


MichaelMcF
Science Freak
MichaelMcF's picture
Posts: 525
Joined: 2008-01-22
User is offlineOffline
Aedus wrote:Any other

Aedus wrote:

Any other explanation than a creator would have the numbers be picked randomly, and do you have any idea how large 10,000 to the 124 is? Scientists already agree that there are variables that are in fact adjusted in our favor despite the fact that there is no reason for them being so. Sure, in the presence of such overwhelming evidence I am forced to draw conclusions. No doubt somebody will respond with something like "just because the odds are high does not mean it's impossible, especially since we exist," but I can always hope someone will respond to my actual argument!

 

2 further comments.  Odds of 1 in 10127 for this universe to form after a big bang event.  How many possible universes where there?  And again you are assuming things are tuned in favour of life and assuming purpose when there's no evidence of any.  We see numbers which support life because we're alive, nothing more.

 

And again, your argument was that a creator is allegedly more sensible than a multiverse and we should defend this.  I'm saying we shouldn't because not all of us think that the multiverse is necessarily required or that an eternal universe is an argument.  Created or Eternal is a false dichotomy to a question which can be answered with "We don't know".  You did admit after all that an agnostic position is an acceptable one.

Forget Jesus, the stars died so that you could be here
- Lawrence Krauss


HisWillness
atheistRational VIP!
HisWillness's picture
Posts: 4100
Joined: 2008-02-21
User is offlineOffline
Aedus wrote:HisWillness

Aedus wrote:
HisWillness wrote:
I am. You're reaching conclusions rather than saying, "it could be this or that". I would agree that it's possible that a creator could have created the universe.

No, I am in fact saying that it could be this or that. In my personal opinion one scenario is more likely than another.

Oh. And you base your opinion on a calculation that is, itself, dubious?

Just so we're clear, let me show you why I think your argument is confusing, and then you can correct me, because you've said a couple of times that nobody is addressing your actual argument, and I haven't been able to really figure out what it is. It SEEMS like it's:

Our universe, with its constants, is unlikely, therefore it was created by an intelligent creator.

Is that it, or not? I don't want to keep missing what you're saying. That seems to be a non-sequitur, as the first part has nothing to do with creators, and the second part has nothing to do with likelihood, and there's nothing to tie them together or address intelligence. If we had:

(1) Our universe is unlikely to exist

(2) All things unlikely to exist are created by an intelligent creator

---

(3) Our universe is created by an intelligent creator

Then at least we'd be using deductive logic. That seems to be what you're going for. But this seems to be treading in fallacy of accident terrritory.

Aedus wrote:
Intentionality is irrelevant.

Then why does the unlikely part mean anything? Are you literally saying, as I've presented as (2), that all unlikely things are created by an intelligence, but that the intelligence doesn't need to be applied to the problem?

Not only that, but what do we call intelligent that has no intentionality? Nothing. There's simply no such thing. Introducing intelligence just makes the creature (for it now must be a biological creature) more complicated.

Aedus wrote:
Click on the links I provided at the beginning of this post to see why this assertion is wrong. Scientists can in fact extrapolate the range of values for these constants...

Other values being extrapolated doesn't really have anything to do with how likely their current values are. If we can imagine something, it doesn't count as a likelihood. Unicorns are a handy example. We already have horses, and some four-legged hooved creatures have horns, so unicorns are fairly likely, right? Well, no evidence for unicorns.

Aedus wrote:
This merry-go-round is simply a delusion that arises from nitpicking bits from my individual posts and ignoring the main argument that I'm presenting.

Let me know if I got the argument right above.

Aedus wrote:
Note that electrons were predicted before they could be measured.

Yes, because the effects of electrons could be measured. Math did the predicting. No such math exists for something like a creator, because it's not clear enough what is being posited by saying "creator".

Aedus wrote:
Any other explanation than a creator would have the numbers be picked randomly, and do you have any idea how large 10,000 to the 124 is?

First, I'm going to be blunt with you: that calculation is ridiculous bullshit. A couple of people have already explained why, but you seem to want to stick with it. Is the idea that a creator's odds are ... what, 100 to 1, then? That everything else is 10 to the 1,255 is arbitrary and based on nonsensical, mad ramblings. Let us know what a creator's odds are of existing.

If I put a creator's odds at 10 to the 1000, would that fix our problem? Because I can invent math for that just like your source did.

Second, numbers picked randomly? Are you suggesting there would be another, different universe? Because then, I suppose a different kind of life would exist. Hey, given that we can suggest just about anything, here, why not? That calculation really just determined how likely our universe was, and not how likely another universe was.

So if a different type of universe, with a different type of life existed, would that still mean a creator?

Aedus wrote:
Scientists already agree that there are variables that are in fact adjusted in our favor despite the fact that there is no reason for them being so.

Be clear. They can think of no reason for them to be so. That does not mean that those variables were specifically adjusted by an intelligence. Here, you've made an appeal to authority, a base rate fallacy, an argument from silence, presented a false dilemma, and further committed a fallacy of necessity. Those are just the ones I can remember. To put it step-by-step:

(1) Scientists can think of no reason that our universe's physical constants are what they are

(2) This universe exists

---

(3) Therefore, an intelligent creator must have created the universe.

So yes, you're giving us a lot of non sequitur. I can't help but notice all the fallacies in your argument. If you want to consider that "nit-picking", then I don't know what to tell you.

Aedus wrote:
Sure, in the presence of such overwhelming evidence I am forced to draw conclusions.

It's safe to say, then, that you're not a scientist. That's not an ad hominem, but your attitude towards evidence is not empirical. Evidence is only useful when it supports a hypothesis, and you have not presented anything resembling a hypothesis. You're reaching a conclusion before you even arrive at a hypothesis, so how can you say you have evidence?

Saint Will: no gyration without funkstification.
fabulae! nil satis firmi video quam ob rem accipere hunc mi expediat metum. - Terence


cervello_marcio
Superfan
cervello_marcio's picture
Posts: 210
Joined: 2009-05-19
User is offlineOffline
Aedus wrote:MichaelMcF

Aedus wrote:

MichaelMcF wrote:
Why does there need to be a reason for these constants to have the values?

Uhm, so that we can live? This article explains it pretty well: http://islam.thetruecall.com/modules.php?name=News&file=article&sid=362

"Do not, as some ungracious pastors do, show me the steep and thorny way to heaven. Whiles, like a puff'd and reckless libertine, himself the primrose path of dalliance treads. And recks not his own rede."


MichaelMcF
Science Freak
MichaelMcF's picture
Posts: 525
Joined: 2008-01-22
User is offlineOffline
EDIT - I can't alter the

EDIT - I can't alter the original post.  This:

Me wrote:

....is that Penrose's number is that it is tied to the probability...

 

Should read "is that Penrose's number is tied to the probability".

Forget Jesus, the stars died so that you could be here
- Lawrence Krauss


HisWillness
atheistRational VIP!
HisWillness's picture
Posts: 4100
Joined: 2008-02-21
User is offlineOffline
MichaelMcF wrote:Aedus

MichaelMcF wrote:

Aedus wrote:

And why not? If anything can be an uncaused cause then wouldn't it be god? You have better ideas? Also, I suggest you familiarize yourself with my actual argument better before you label it as "special pleading". I've seriously explained my stance like a dozen times in this thread.

But it is special pleading.  You assume that the universe must have a cause.  You then say something must have caused it and this uncaused cause is god.  I ask you why the universe can't be an uncaused caused.  To say that god can be an uncaused cause but the universe can't is special pleading.

It's the textbook example of special pleading. To put the rhetorical question above in the form of a statement, "if anything can be an uncaused cause, then it would be god".

The real problem, however, with introducing something that doesn't need to be caused is that later, you can't come back and say that everything needs a cause. We can no longer say that everything has a cause, and that therefore the universe has a cause. Because we know of exactly one thing that doesn't need a cause, so we then know that not everything needs to be caused, including the universe.

You give an example of another fallacy of silence with "you have better ideas"? Implying that in the absence of a better idea, an idea that is based on a fallacy is superior to none at all.

Saint Will: no gyration without funkstification.
fabulae! nil satis firmi video quam ob rem accipere hunc mi expediat metum. - Terence


Aedus
Posts: 33
Joined: 2009-08-07
User is offlineOffline
MichaelMcF wrote:The problem

MichaelMcF wrote:
The problem with this (if it is true - still trying to get my hands on the original material) is that Penrose's number is that it is tied to the probability of the universe occupying the state it's in after a big bang event.  This has nothing to do with creation or "origin" so you can stop throwing that number around.

Luckily the # is not really the focal point of my argument. Read the rest of my argument here (the first section of large paragraphs). The other origin explanations are also problematic.

 

MichaelMcF wrote:
No, this is not nitpickery.  You are assuming creation to come to the conclusion of creation.  It's circular logic. 

Then change "creation" to "the universe".

Quote:
Your main point also assumes that the default position is a multiverse theory when that is blatantly untrue.

Hence the reason I provided alternatives: that the universe was created or it is accidental. It can only be one of the three. Do you have any more options?

Quote:
The Planck constant is 6.63x10-34.  You're arguing that if the number was altered by 1x10-8 then things would be different.  This number is 1x1025 times larger than Planck's constant.  That's quite a large difference you're asking for there.  That's the difference between the weight of a bag of sugar and almost 2 times the weight of the earth.  On that kind of scale I'm not surprised that changing Planck's number would have strange effects!

First off, we're not making it clear what units we're using. The planck constant is 6.63x10-34 joule-seconds but it's 6.6 * 10 -23 erg-seconds, which is what I was referring to (so, my bad). Second, the point is that there is no reason why it should not have been higher by that amount.  

Quote:
McDonalds has a creator (as a company, a franchise and a building) that we can tie down as distinctly human, thus there is no reason to assume that we'll find McDonalds on other planets.

Yes, just like life exists only on this planet - so we can assume that it doesn't exist on other planets, right?

Quote:
Your borg cube theory also only works if it is assumed that alien life must be borg like entities intent on destroying the white house.  If we're just talking about the logical possibility of life having evolved elsewhere then you're argument falls apart as it holds absolutley no connection to the base discussion.

Exactly! You're now my new favorite poster! Laughing out loud

See, all I'm talking about is the logical possibility of the universe having been created instead of being eternal or an accident. Yet you guys insist that my "creator" must have many more properties than is necessary and that he must be proven beyond a shadow of a doubt to lend him any credence. Therefore I will demand the same thing of ET life. Smiling

Quote:
This is ridiculous and you know it.

I do know it - but it's uncannily representative of the logic that atheists sometimes use.

 

Quote:
Certainly

I don't see any part that actually says that proteins were found on meteorites. This was an experiment which recreated ten of the 20 amino acids used in proteins and the article suggests that the remainder could have been delivered to Earth via meteorites.

HisWillness wrote:
Oh. And you base your opinion on a calculation that is, itself, dubious?

It's not dubious. We've been over this. <_<

Even IF the calculation itself is dubious, the #s are still high & my argument still stands.

HisWillness wrote:
Just so we're clear, let me show you why I think your argument is confusing, and then you can correct me, because you've said a couple of times that nobody is addressing your actual argument, and I haven't been able to really figure out what it is. It SEEMS like it's:

Our universe, with its constants, is unlikely, therefore it was created by an intelligent creator.

Is that it, or not? I don't want to keep missing what you're saying. That seems to be a non-sequitur, as the first part has nothing to do with creators, and the second part has nothing to do with likelihood, and there's nothing to tie them together or address intelligence. If we had:

(1) Our universe is unlikely to exist

(2) All things unlikely to exist are created by an intelligent creator

---

(3) Our universe is created by an intelligent creator

Then at least we'd be using deductive logic. That seems to be what you're going for. But this seems to be treading in fallacy of accident terrritory.

That's basically it, except #2 is not as affirmative a statement as you wrote. I am merely comparing all the possible alternatives & choosing one based on parsimony. And it doesn't necessarily have to be a creator, just the idea that our universe was "created".

Quote:
Then why does the unlikely part mean anything? Are you literally saying, as I've presented as (2), that all unlikely things are created by an intelligence, but that the intelligence doesn't need to be applied to the problem?
Even if it was an accident, it does not mean that intelligence was not applied to the problem. Perhaps life is an inadvertent effect of the creator's machinations?

 

Quote:
Other values being extrapolated doesn't really have anything to do with how likely their current values are. If we can imagine something, it doesn't count as a likelihood. Unicorns are a handy example. We already have horses, and some four-legged hooved creatures have horns, so unicorns are fairly likely, right? Well, no evidence for unicorns.

So you disagree that my conclusion follows from my premises? What conclusions would you draw then for the universe existing yet being so unlikely to exist? Other than "we lucked out" or "I don't know". If it's the former, then why is that more probable than the fact that our universe could have been created? If it's the latter, then how can you say for certainty that it's not more likely for the universe to have had a creator?

Quote:
Yes, because the effects of electrons could be measured. Math did the predicting. No such math exists for something like a creator, because it's not clear enough what is being posited by saying "creator".

Hmm, it seems that you disagree that this kind of chance points to design. You find it inconceivable that an intelligence could have picked the numbers out, but you don't find it inconceivable that they all just randomly found themselves that way for no apparent reason. Evidence of design can be found in any kind of universe - this is backed up in one of the articles I linked to earlier that discussed the probabilities.

 

Quote:
First, I'm going to be blunt with you: that calculation is ridiculous bullshit. A couple of people have already explained why, but you seem to want to stick with it.

No, they haven't. Even if you disagree with the calculation, the number is still absurdly high. Now you're just being difficult and are attempting to argue against years of established cosmological science.

Quote:
Is the idea that a creator's odds are ... what, 100 to 1, then? That everything else is 10 to the 1,255 is arbitrary and based on nonsensical, mad ramblings. Let us know what a creator's odds are of existing.

If I put a creator's odds at 10 to the 1000, would that fix our problem? Because I can invent math for that just like your source did.

It's too bad that I'm not using numbers, but the amount of assumptions. In the universe's case, it's all the variables that have to fit just right but can have a wide range of values. It's called Occam's Razor.

 

Quote:
Be clear. They can think of no reason for them to be so. That does not mean that those variables were specifically adjusted by an intelligence.

Yet again, what is a better explanation and why are the alternatives more likely? When you see constants just fine-tuned like that, fine-tuned as if by something that knew what it was doing, I think that they were adjusted by an intelligence is really the only obvious conclusion. But I'm not interested in arguing personal preferences.

 

Quote:
It's safe to say, then, that you're not a scientist. That's not an ad hominem, but your attitude towards evidence is not empirical. Evidence is only useful when it supports a hypothesis, and you have not presented anything resembling a hypothesis. You're reaching a conclusion before you even arrive at a hypothesis, so how can you say you have evidence?

What are you talking about dude? Not all evidence has to be empirical, even scientific evidence. Have you ever read or looked at any speculative science like theoretical physics, etc? Again, applying your kind of logic to everyday matters would be ridiculous and it would be like saying that the existence of ET life is superstitious & unreasonable just because we have no evidence for it.

HisWillness wrote:
You give an example of another fallacy of silence with "you have better ideas"? Implying that in the absence of a better idea, an idea that is based on a fallacy is superior to none at all.

There are better ideas - I've highlighted all the possible ones in my main post (except "accidental" since this forum software won't let you edit old posts). You guys keep telling me that "atheists don't subscribe to an eternal universe!" so what exactly is your position on the subject? You can't plead ignorance here & say that we don't have enough knowledge, because we most certainly know enough to determine what is more reasonable/probable with our current knowledge of multiverses & cosmology. Remember, an atheist universe IS possible, it just makes some serious assumptions based on current science. And as I said before, an agnostic attitude is also fine to take, in which case you admit that either scenario is as likely at this point.

HisWillness wrote:
It's the textbook example of special pleading. To put the rhetorical question above in the form of a statement, "if anything can be an uncaused cause, then it would be god".

As I said before, take God out of the equation if you need to. Do you admit that a created universe is just as likely, if not more likely than the other explanations?


Vastet
atheistBloggerSuperfan
Vastet's picture
Posts: 13234
Joined: 2006-12-25
User is offlineOffline
How amusing that you try

How amusing that you try applying Occam to us without doing the same to yourself.
Universe = Pretty damn complex.
God = More so.
Simplest answer: No god.

Enlightened Atheist, Gaming God.


latincanuck
atheist
latincanuck's picture
Posts: 2038
Joined: 2007-06-01
User is offlineOffline
Le me get this straight

 If I can undrestand your logic is the following, So far a naturally explaining universe, because we have for the most part a natural occuring explaination for most things in the universe (again we haven't defined all things in the universe, yet) so you need a supernatural, undefinable creator to explain a natural world....and that's basic logic? Are you kidding me? Please again explain to me your basic logic? That all naturally occuring things (planets naturally forming, sun naturally forming, natural explaination for the atmosphere occuring, natural explaination for the evolution of all the creatures to date. Natural explaination of so many things in this universe, yet somehow you need a supernatural deity that defys explaination to explain the universe?


HisWillness
atheistRational VIP!
HisWillness's picture
Posts: 4100
Joined: 2008-02-21
User is offlineOffline
Aedus wrote:HisWillness

Aedus wrote:

HisWillness wrote:
Oh. And you base your opinion on a calculation that is, itself, dubious?

It's not dubious. We've been over this. <_<

Even IF the calculation itself is dubious, the #s are still high & my argument still stands.

Haha! Oh, I see. You've been joking this whole time. I fell for it, I admit it.

"The calculation is dubious, but even if it's off, my argument still stands." Haha! That's a good one. You got me.

Aedus wrote:
That's basically it, except #2 is not as affirmative a statement as you wrote. I am merely comparing all the possible alternatives & choosing one based on parsimony. And it doesn't necessarily have to be a creator, just the idea that our universe was "created".

Rich. The universe was "created", but there's no creator now? Despite the idea that the creator earlier presented was intelligent? I can't believe you sucked me in. I'm so gullible.

Aedus wrote:
So you disagree that my conclusion follows from my premises?

It's this kind of thing that should have made it so clear. There I was, identifying four or five different fallacies by heart, and you were just stringing me along. You have your victory, sir, I was fooled.

Aedus wrote:
Hmm, it seems that you disagree that this kind of chance points to design. You find it inconceivable that an intelligence could have picked the numbers out, but you don't find it inconceivable that they all just randomly found themselves that way for no apparent reason. Evidence of design can be found in any kind of universe - this is backed up in one of the articles I linked to earlier that discussed the probabilities.


How did you come up with this character? It's brilliant! I mean honestly, take a bow. The chance involved in ... measuring electrons? You're amazing. I'm a fan.

Aedus wrote:
No, they haven't. Even if you disagree with the calculation, the number is still absurdly high. Now you're just being difficult and are attempting to argue against years of established cosmological science.


Hahahaha! That's fantastic. I seriously love that line. Even if the calculation's wrong, the number is still high. So good.

Aedus wrote:
It's too bad that I'm not using numbers, but the amount of assumptions. In the universe's case, it's all the variables that have to fit just right but can have a wide range of values. It's called Occam's Razor.


That one is gold. If you use that in a routine (like stand-up at a scientific convention), leave that for the big finale. You'll kill. Just keep saying Occam's Razor when it doesn't apply, like "that's what she said".

Aedus wrote:
What are you talking about dude? Not all evidence has to be empirical, even scientific evidence. Have you ever read or looked at any speculative science like theoretical physics, etc? Again, applying your kind of logic to everyday matters would be ridiculous and it would be like saying that the existence of ET life is superstitious & unreasonable just because we have no evidence for it.


You were born for this. How many other atheist sites have you gone to? Scientific evidence not being empirical. I love it.
Aedus wrote:
As I said before, take God out of the equation if you need to. Do you admit that a created universe is just as likely, if not more likely than the other explanations?

I admit that you totally got me. I really thought you were serious. Look at all the other guys here you suckered, too. That was masterful.

Saint Will: no gyration without funkstification.
fabulae! nil satis firmi video quam ob rem accipere hunc mi expediat metum. - Terence


cervello_marcio
Superfan
cervello_marcio's picture
Posts: 210
Joined: 2009-05-19
User is offlineOffline
 @willaedus wrote:indeed,

 @will

aedus wrote:

indeed, going by atheist logic I'll go ahead & say that unless a borg cube holds orbit and destroys the white house then it's unreasonable & superstitious to think that ET life exists. 

the return of treat2?

"Do not, as some ungracious pastors do, show me the steep and thorny way to heaven. Whiles, like a puff'd and reckless libertine, himself the primrose path of dalliance treads. And recks not his own rede."


HisWillness
atheistRational VIP!
HisWillness's picture
Posts: 4100
Joined: 2008-02-21
User is offlineOffline
cervello_marcio

cervello_marcio wrote:

 @will

aedus wrote:

indeed, going by atheist logic I'll go ahead & say that unless a borg cube holds orbit and destroys the white house then it's unreasonable & superstitious to think that ET life exists. 

the return of treat2?

 

Oh man, that would be a double-whammy!

PS - Hey Aedus, are you in Dembski's class?

Saint Will: no gyration without funkstification.
fabulae! nil satis firmi video quam ob rem accipere hunc mi expediat metum. - Terence


MichaelMcF
Science Freak
MichaelMcF's picture
Posts: 525
Joined: 2008-01-22
User is offlineOffline
This is the second time I've

This is the second time I've had to write this, as I accidentally closed my browser first time and Chrome doesn't have a "are you sure you want to shut down you clown?" fail-safe.  So I apologise for any mistakes I make in the rush.

Aedus wrote:

Luckily the # is not really the focal point of my argument. Read the rest of my argument here (the first section of large paragraphs). The other origin explanations are also problematic.

I didn't say it was the focal point, but it did seem important enough for you to mention it several times.  If you want to drop it it's fine (although as you mentioned that this number is "practically" zero I'd still like to hear your thoughts on the odds of a particular order of a deck of cards being 1 in 8x1067 and whether this is an impossible event or not...)

 

So I'll address your original argument which I'm reading as thus:

Aedus wrote:

The evidence is [the universe], the conclusion is a creator. I make no claims about any of his properties, other than that he is the uncaused cause. We know that simple matter cannot be the cause of its own existence, so we must find something that can. My conclusion follows from the evidence because there are many laws of [the universe] which defy any natural need for their existence... ...There is no reason for these constants to be what they are. The odds of such a universe forming are so small that they should basically be zero, which points to a creator. It's deductive reasoning.

You've yet to explain why the universe needs a cause.  Why does this creator get a free pass but the universe doesn't?

Secondly, how do you know the numbers can be different?  I'm not being difficult here.  How do you know the numbers could have been different?  This whole line of reasoning is predicated on the assumption that the formation of the universe during and after a big bang event is a totally random process.  How do you know this?  For all we know the numbers we've observed are the only possible outcome.  Without seeing other verses forming we have no way of knowing.

You're like a guy looking at a photograph taken 100 years ago which shows the top, and only the top,  of 17 die all showing sixes and saying "There's only a 1 in 1010* chance that that happened randomly.  Somone must have put them there!".  With no way of witnessing the original event you've got no way of knowing the that die didn't have sixes on every single side.

Bear in mind that I'm not arguing that there is only one way the numbers could have come up.  I'm simply asking how you know that they could have been different.  If you have some way of demonstrating that the numbers are random I'm sure there are a few physicists that would like to hear it.

 

 

Aedus wrote:

Hence the reason I provided alternatives: that the universe was created or it is accidental. It can only be one of the three. Do you have any more options?

What I actually have is a refinement of your statements.  You're claiming that there are only three options for the existence of the universe; Creation, Accident or Eternal.  I don't like the word 'accident'.  Accident implies blame or inadvertent cause, and thus a creator as already demonstrated by Saint Will.  So we're back to two.

I think the three options that you're trying to get at are: Created (with seemingly ambiguous intent), Eternal or Had Beginning (the old something from nothing).  You're also setting up a false argument that our position must be one of these three and thus we should be prepared to defend it.  Let me break it down for you.

  • Created.  Based purely on textbook special pleading.  The introduction of a creator asks the question "what created the creator" ad infinitum.  The only way to stop this is to give a special clause to your position which you have no good reason for not applying to other positions.  It creates more assumptions and thus can be disposed of thanks to Occam's razor.
  • Eternal.  Often not an argument but rather a question against the aforementioned special pleading - "Why does the creator get to be eternal but the universe doesn't?".  If taken as an actual positions then we have a myriad of possibilites from the multi or cyclic verse theories.  As we have no way of knowing what happened before the big bang these things have as much evidence for them as the universe being farted into existence and thus "I don't know" is a more sensible position.
  • Had Beginning.  Somewhere in-between the other two, this is based on the assumption that the universe had a beginning but no real cause.  The "popped-out-of-nothing" argument.  A little bit of a strawman argument because it isn't really a scientific position (big bang theory isn't about the creation of the universe, nor does it say something came from nothing) and suffers from the same flaw as the other two - we have no way of seeing beyond the big bang so anything we say about it is wild conjecture.  It is much simpler to say "I don't know".

So tell me, why should I defend any of these positions when I think "I don't know" is logically the best answer?

 

Aedus wrote:

First off, we're not making it clear what units we're using. The planck constant is 6.63x10-34 joule-seconds but it's 6.6 * 10 -23 erg-seconds, which is what I was referring to (so, my bad). Second, the point is that there is no reason why it should not have been higher by that amount.

Well excuse me, but as you mentioned the number but didn't write it down or specify units I took the accepted route of assuming you were following SI convention.

So we're going with 6.6x10-23 then?  And your saying a change of 1x10-8 in this number would cause the universe to be different?  A number 1x1014 times larger than the constant itself?  So you're argument is now "if a bag of sugar was as massive as all the biomass on earth (thank you wolframalpha...) it would be heavier!".  Amazing but not profound or insightful.

I once again ask you to explain how you know the number could have been different.

 

Aedus wrote:

Yes, just like life exists only on this planet - so we can assume that it doesn't exist on other planets, right?

Wrong.  Your statement is a bold assertion with no evidence.   We do not know that life only exists on this planet.  That is completely untrue.

 

Aedus wrote:

Exactly! You're now my new favorite poster! Laughing out loud

See, all I'm talking about is the logical possibility of the universe having been created instead of being eternal or an accident. Yet you guys insist that my "creator" must have many more properties than is necessary and that he must be proven beyond a shadow of a doubt to lend him any credence. Therefore I will demand the same thing of ET life. Smiling

Wrong.  Again.  I haven't introduced any further properties or asked for anything beyond a shadow of a doubt.  I ask two things of you, and they are standards that I apply to any scientific hypothesis:

1.  Explain why any property you assign to your system (uncaused cause) cannot be applied to other systems.

2.  Provide any evidence that strongly suggests - not proves - that the observations can only be explained by your hypothesis.

As you've yet to explain why the creator gets to be an uncaused cause and the universe doesn't, and all observations of the universe can be explained using current, standard theories in science you have failed on both counts.

 

Aedus wrote:

I don't see any part that actually says that proteins were found on meteorites. This was an experiment which recreated ten of the 20 amino acids used in proteins and the article suggests that the remainder could have been delivered to Earth via meteorites.

Excuse me?  Proteins?  Who mentioned proteins?  Amino acids are the building blocks of life my friend.  If I was being a dick I'd say carbon is the building block of life but I thought I'd stick with the simplest molecular unit.  Why did you assume proteins?

I've highlighted the above section because it proves that you didn't read the article or, if you did, you didn't understand it.  The article does not suggest that 10 amino acids were created here and then the rest were brought by a meteorite.  It doesn't suggest this in the slightest.  I'll break this down for you too:

  • The article states that in almost all synthesis of amino acids - hydrothermal, primordial soup or interstellar - we see roughly the same proportions of 10 amino acids forming.
  • These proportions match the numbers you get if you try and predict things thermodynamically.  In other words the proportions we see match those we'd expect to see forming in a naturalistic system.
  • Furthermore these 10 amino acids form under such a variety of conditions that there is no reason that they couldn't form anywhere in the universe.
  • Given the commonality of these 10 amino acids and the fact they are more thermodynamically likely than all the others, it is not unreasonable to assume that early proteins and life may have formed using nothing but these 10 acids and that the other 10 in our biological frame were introduced by catalytic/enzymatic processes further down the evolutionary road.
  • It is then also not unreasonable to assume that the same processes could have happened elsewhere in the universe.  The thermodynamic ease of formation of these acids precludes the idea that life can only exist on our little world.

So yes, the building blocks of life can exist and form elsewhere in the universe.

 

Aedus wrote:

Even IF the calculation itself is dubious, the #s are still high & my argument still stands.... [and] .... No, they haven't. Even if you disagree with the calculation, the number is still absurdly high.

If the calculation is dubious then the validity of the numbers is questionable.  You'd be as well using numbers that you plucked out your ass... a high number doesn't mean anything unless it was reached through sensible means.  If the calculation is dubious so is the number.

But then you've told me it's not the focal point of your argument so it's not like it matters right?  Right?

 

 

*{EDIT:  This should be (roughly) 1 in 1013}

Forget Jesus, the stars died so that you could be here
- Lawrence Krauss


Aedus
Posts: 33
Joined: 2009-08-07
User is offlineOffline
HisWillness - is this

HisWillness - is this already the point of the debate where you guys start getting condescending/hostile?

*grabs popcorn

Either way, simply rattling off a list of fallacies does not mean that I've made them, and you haven't shown that I have. It's hard to argue with someone who doesn't back up his statements. Your list of fallacies is entirely dependent on the idea that my obvious conclusion doesn't follow the premises. There are many people, atheist scientists included, who agree that the fine-tuning of these constants points to a design. That is the entire of point of creating cyclical universe theories - to eliminate these odds.

The calculation - I've provided the name of the person and institute that initiated this study. You guys have provided nothing apart from saying that it is wrong/suspect without any reasoning/proof. I've also provided a list of references from atheists/agnostics who discuss these numbers.

MichaelMcF wrote:
You've yet to explain why the universe needs a cause.  Why does this creator get a free pass but the universe doesn't?


We know the universe had a definite beginning - this has been shown by the second law of thermodynamics. We know that matter can't be the instrument of its own creation (excluding quantum vacuum & cyclical universe theories). To the best of our knowledge, matter DID have a beginning - its volume was zero at the very beginning of the big bang.
The idea of a creator makes way fewer assumptions. This is the entire reason that scientists have been dabbling in cyclic universe theories like string theory!

Quote:
Secondly, how do you know the numbers can be different?


Because it's already been shown by various cosmologist that they could have been different. For people that apparently hold science on a pedestral, you guys sure do like to disagree with it alot.

Quote:
So tell me, why should I defend any of these positions when I think "I don't know" is logically the best answer?


As I've said like 10x, you're free to go with I don't know, in which case I doubt that would make you a true atheist. And let's face it, if it were really your answer, you wouldn't be here arguing with me about it. In fact, I wonder how many real agnostics are on these boards?

Quote:
Wrong.  Your statement is a bold assertion with no evidence.   We do not know that life only exists on this planet.  That is completely untrue.


Just like the atheist position is a bold assertion with no evidence. We do not know that God does not exist.
Quote:
Excuse me?  Proteins?  Who mentioned proteins?  Amino acids are the building blocks of life my friend.  If I was being a dick I'd say carbon is the building block of life but I thought I'd stick with the simplest molecular unit.  Why did you assume proteins?

I've highlighted the above section because it proves that you didn't read the article or, if you did, you didn't understand it.  The article does not suggest that 10 amino acids were created here and then the rest were brought by a meteorite.  It doesn't suggest this in the slightest.  I'll break this down for you too:

    * The article states that in almost all synthesis of amino acids - hydrothermal, primordial soup or interstellar - we see roughly the same proportions of 10 amino acids forming.
    * These proportions match the numbers you get if you try and predict things thermodynamically.  In other words the proportions we see match those we'd expect to see forming in a naturalistic system.
    * Furthermore these 10 amino acids form under such a variety of conditions that there is no reason that they couldn't form anywhere in the universe.
    * Given the commonality of these 10 amino acids and the fact they are more thermodynamically likely than all the others, it is not unreasonable to assume that early proteins and life may have formed using nothing but these 10 acids and that the other 10 in our biological frame were introduced by catalytic/enzymatic processes further down the evolutionary road.
    * It is then also not unreasonable to assume that the same processes could have happened elsewhere in the universe.  The thermodynamic ease of formation of these acids precludes the idea that life can only exist on our little world.

So yes, the building blocks of life can exist and form elsewhere in the universe.


Aside from the fact that you're wrong about the building blocks (it is in fact proteins, lipids, carbs, and nucleic acids that are the building blocks of life, not amino acids), this article does not show anywhere that these things have been found in meteorites, merely proposing it. I didn't ask you if it was probable that it could form.

Vastet wrote:
How amusing that you try applying Occam to us without doing the same to yourself. Universe = Pretty damn complex. God = More so. Simplest answer: No god.

Very well then. The following list is the three possible ways our universe came to be and the assumptions that go into them. If anybody has anything to add/edit/remove then feel free to do so - I don't want anybody claiming that I'm building a strawman here, though no doubt somebody will pipe up with that assertion.

The universe is either:

- Accidental (the stupid "what if this is the only universe" argument)

  • Assumes that matter is the instrument of its own creation despite there being no reason for it. Matter had a volume of zero during start of big bang.
  • Violates second law of thermodynamics (unless the universe is cyclic, but this isn't that).
  • Protons and electrons have the same exact charge of 1.6-19 coulombs despite the fact that a proton has a mass 1836 times that of an electron. There is no natural reason for these #s to be the same and if they weren't our universe wouldn't have lasted as long due to stars using up their thermonuclear fire.
  • If the strong force had been any more intense during first few minutes of universe's expansion (trillionth of a centimeter change), all of the universe's hydrogen would have been converted to helium & life wouldn't exist. There is no reason for this not to have been the case.
  • Planck Constant is 6.6-23 erg/second. Had it been different by .00000001 (no natural reason to) there would be no life.
  • Speed of Light is 3e8 m/s. If it was a little higher then stars would die out too soon, if it was a little lower then the universe would have suffered gravitational collapse (again, no natural reason for it to have been different).
  • If the neutron mass was 2% greater than what it is then neutrons would rapidly decay and all atoms would be unstable, and if it was a little lower then protons would become unstable. Again, no natural reason for them not to.

*Note - things such as the flatness problem or the isotropy problem also had inexoricably high odds, but they had NATURAL reasons for being the way they were so I've left them out. The things above have no reason to be the way they are.

- Cyclic (quantum fluctuations, brane collisions, etc. - make the largest assumptions)

  • Each diff theory has its own set of assumptions, none based on empirical evidence.
  • The basic premise is to create an infinite number of universes just to explain the existence of this one (not sure how much bigger of an assumption it's possible to have...)
  • This assumes that the multiverse, with all of its intricacy & complexity existed for eternity.
  • It assumes that such a complex thing as the fabric of space-time existed exactly the way it does for eternity for no apparent reason, which allows for things such as gravity, motion, etc.
  • It assumes the problem of crossing infinities, a futile task. How far do you have to walk across infinity to get to "now"? Is that even possible?
  • Just because something is infinite does not mean it will generate the correct thing. Suppose our universe's number is 1.56, but there is also an infinite amount of numbers between 0 and 1. What if the multiverse only generates those numbers? The multiverse theories assume that a universe with life is in the range for them to generate.


- Created (fewest assumptions)

  • The universe was created.
  • The creator is the uncaused cause.
  • The creator has an intelligence that he applied to the constants.

I want rational/scientific reasons for why you guys think that the first two options are more likely than the second.